23.10.2014 Views

Cardiology/Endocrinology - ACCP

Cardiology/Endocrinology - ACCP

Cardiology/Endocrinology - ACCP

SHOW MORE
SHOW LESS

Create successful ePaper yourself

Turn your PDF publications into a flip-book with our unique Google optimized e-Paper software.

Answers to PSAP 2013 Book 1<br />

(<strong>Cardiology</strong>/<strong>Endocrinology</strong>)<br />

<strong>Cardiology</strong> I<br />

Acute Decompensated Heart Failure<br />

Questions 1–3 pertain to the following case.<br />

H.M. is a 57-year-old woman with ischemic cardiomyopathy<br />

(ejection fraction [EF] 30%) who presents to the<br />

emergency department with dyspnea, and fatigue. Her vital<br />

signs include blood pressure (BP) 90/75 mm Hg, heart rate<br />

92 beats/minute, respiratory rate 22 breaths/minute, and<br />

oxygen saturation 95% on 4 L nasal cannula. Physical examination<br />

reveals a 15-cm jugular venous distention ( JVD),<br />

crackles bilaterally, regular rhythm and rate (RRR), and 3+<br />

pitting edema. Pertinent laboratory values include sodium<br />

126 mmol/L, potassium 3.9 mmol/L, BUN 50 mg/dL, and<br />

SCr 2.1 mg/dL (baseline 1.8 mg/dL). H.M.’s drugs include<br />

lisinopril 20 mg/day, carvedilol 12.5 mg twice daily, furosemide<br />

40 mg twice daily, and spironolactone 25 mg/day.<br />

She is adherent to both dietary restrictions and her drugs.<br />

1. H.M.’s signs and symptoms are most consistent with<br />

which one of the following?<br />

A. Low cardiac output (CO) only.<br />

B. Fluid overload only.<br />

C. Both fluid overload and low CO.<br />

D. Neither fluid overload nor low CO.<br />

1. Answer: C<br />

The presenting signs and symptoms of ADHF are generally<br />

classified as symptoms of fluid overload or low cardiac output.<br />

Pulmonary symptoms (e.g., dyspnea) and signs (e.g.,<br />

wheezing, hypoxemia) are associated with elevated pulmonary<br />

filling pressures and pulmonary capillary wedge<br />

pressure (PCWP). Peripheral edema and ascites are the<br />

most common signs of systemic fluid overload. Signs and<br />

symptoms of low cardiac output result in end-organ hypoperfusion,<br />

which can be evidenced by elevated SCr, elevated<br />

hepatic transaminases, and gut ischemia manifested by nausea,<br />

vomiting, abdominal pain and early satiety. This patient<br />

shows signs and symptoms of both fluid overload and low<br />

cardiac output, as evidenced through her complaints and<br />

physical examination (Answer C is correct; and Answer A,<br />

Answer B, and Answer D are incorrect).<br />

1. Rodgers JE, Lee, CR. Acute decompensated heart failure. In:<br />

DiPiro JT, Talbert RL, Yee G, et al., eds. Pharmacotherapy: A<br />

Pathophysiologic Approach, 8th ed. New York: McGraw-Hill,<br />

2011:173-216.<br />

2. Joseph SM, Cedars AM, Ewald GA, et al. Acute decompensated<br />

heart failure. Tex Heart Inst J 2009;36:510-20.<br />

PubMed Link<br />

2. Which one of the following is the best predictor of<br />

H.M.’s risk of in-hospital mortality?<br />

A. B-type natriuretic peptide (BNP).<br />

B. BUN.<br />

C. SCr concentration.<br />

D. Systolic BP.<br />

2. Answer: B<br />

Data from the Acute Decompensated Heart Failure<br />

National Registry (ADHERE) analyzed 39 variables and<br />

found the strongest discriminator of in-hospital mortality<br />

to be an elevated blood urea nitrogen (BUN) of 43 mg/dL<br />

or higher (Answer B is correct). B-type natriuretic peptide<br />

(BNP), SCr, and systolic BP can also be used to assess risk;<br />

however, their association was not as strong as that of BUN<br />

(Answer A, Answer C, and Answer D are incorrect).<br />

1. Fonarow GC, Adams KF, Abraham WT, et al. Risk stratification<br />

for in-hospital mortality in acutely decompensated heart<br />

failure. JAMA 2005;293:572-80.<br />

PubMed Link<br />

2. Adams KF, Fonarow GC, Emerman CL, et al. Characteristics<br />

and outcomes of patients hospitalized for heart failure in the<br />

United States: rationale, design, and preliminary observations<br />

from the first 100,000 cases in the Acute Decompensated<br />

Heart Failure National Registry (ADHERE). Am Heart J<br />

2005;149:209-16.<br />

PubMed Link<br />

PSAP 2013 • <strong>Cardiology</strong>/<strong>Endocrinology</strong> 1 Answers


3. Which one of the following best explains H.M.’s<br />

decompensation?<br />

A. Dietary nonadherence.<br />

B. Kidney insufficiency.<br />

C. Acute arrhythmia.<br />

D. Progression of heart failure (HF).<br />

3. Answer: D<br />

This patient reports dietary adherence (Answer A is incorrect).<br />

Although her SCr is elevated, this is not a significant<br />

increase and is likely an indication that her HF is progressing<br />

(Answer B is incorrect). The patient’s ECG shows<br />

normal sinus rhythm, and she did not have symptoms of<br />

recent palpitations (Answer C is incorrect). The patient<br />

does exhibit hyponatremia, which is suggestive of worsening<br />

HF (Answer D is correct).<br />

1. McMurray JJ, Adamopoulos S, Anker SD, et al. European<br />

Society of <strong>Cardiology</strong> guidelines for the diagnosis and<br />

treatment of acute and chronic heart failure. Eur Heart J<br />

2012;33:1787-847.<br />

PubMed Link<br />

2. Lindenfeld J, Albert NM, Boehmer JP, et al. Heart Failure<br />

Society of America 2010 comprehensive heart failure practice<br />

guidelines. J Card Fail 2010;16:e1-e194.<br />

PubMed Link<br />

Questions 4–6 pertain to the following case.<br />

K.T. is a 78-year-old man with nonischemic cardiomyopathy<br />

(EF 25%–30%) who presents to the emergency<br />

department with an acute HF exacerbation. His vital signs<br />

include BP 135/85 mm Hg, heart rate 98 beats/minute,<br />

respiratory rate 24 breaths/minute, and oxygen saturation<br />

96% on 3 L nasal cannula. Physical examination reveals a<br />

14-cm JVD, RRR, crackles bilaterally, and 3+ bilateral lower<br />

extremity edema. K.T. has gained 20 lb in the past 3 weeks;<br />

he reports strict adherence to both dietary restrictions and<br />

medications. He admits smoking 4 or 5 cigarettes per day.<br />

He states his β-blocker dose was increased last month to<br />

obtain better rate control. In the emergency department,<br />

he has already received furosemide 40 mg intravenously x<br />

1 with minimal response in urine output. Pertinent laboratory<br />

values include potassium 4.2 mmol/L, BUN 38 mg/<br />

dL, and SCr 1.5 mg/dL (baseline 1.2 mg/dL). Cardiac<br />

enzymes are within normal limits x 2 sets, and electrocardiography<br />

shows normal sinus rhythm. K.T.’s drugs include<br />

lisinopril 40 mg once daily, metoprolol XL 100 mg once<br />

daily, and furosemide 40 mg twice daily.<br />

4. Given the predicted BNP result, which one of the following<br />

best describes K.T.’s condition?<br />

A. Significant volume overload and ventricular wall<br />

stretch.<br />

B. Active myocardial ischemia.<br />

C. Shortness of breath caused by a noncardiac<br />

etiology.<br />

D. Kidney insufficiency.<br />

4. Answer: A<br />

B-type natriuretic peptide (BNP) is released and elevated<br />

in the setting of significant volume overload, causing stretch<br />

of the ventricular wall. This patient likely has elevated BNP<br />

because he demonstrates multiple signs and symptoms<br />

of volume overload and ventricular stretch (Answer A is<br />

correct). Common laboratory values for assessing active<br />

myocardial ischemia include creatinine kinase, creatinine<br />

kinase-myocardial fraction, and troponin, all of which are<br />

within normal limits (Answer B is incorrect). The BNP may<br />

be used to rule out other etiologies of shortness of breath<br />

due to a noncardiac etiology. However, Answer C is incorrect<br />

because the BNP concentration will be normal in this<br />

situation. Although BNP may be altered in the setting of<br />

renal insufficiency, it is not to the same degree as the concentration<br />

of elevations, which occurs in the setting of fluid<br />

overload (Answer D is incorrect).<br />

1. Januzzi JL, Camargo CA, Anwaruddin S, et al. The N-terminal<br />

pro-BNP investigation of dyspnea in the emergency department<br />

(PRIDE) study. Am J Cardiol 2005;95:948-54.<br />

PubMed Link<br />

2. Weintraub NL, Collins SP, Pang PS, et al. Acute heart failure syndromes:<br />

emergency department presentation, treatment, and<br />

disposition: current approaches and future aims. Circulation<br />

2010;122:1975-96.<br />

PubMed Link<br />

5. Which one of the following would be best to recommend<br />

regarding K.T.’s metoprolol XL dosage?<br />

A. Continue with no changes.<br />

B. Increase to 150 mg once daily.<br />

C. Decrease to 50 mg once daily.<br />

D. Discontinue.<br />

5. Answer: C<br />

The patient experienced fluid accumulation in association<br />

with recent up-titration of his β-blocker dose; therefore,<br />

continuation of this same dose, metoprolol XL 100 mg<br />

once daily, is inappropriate (Answer A is incorrect). An<br />

increase to metoprolol XL 150mg once daily would only<br />

be warranted if the patient was tolerating the current dose<br />

with no fluid accumulation or worsening HF (Answer B is<br />

incorrect). A decrease to metoprolol XL 50 mg once daily,<br />

the previously tolerated dose, would be indicated in this situation<br />

(Answer C is correct). The decision to discontinue<br />

metoprolol XL should rarely occur and only in the setting of<br />

cardiogenic shock (Answer D is incorrect).<br />

1. Jondeau G, Neuder Y, Eicher JC, et al. B-CONVINCED: beta<br />

blocker continuation versus interruption in patients with<br />

Answers<br />

2<br />

PSAP 2013 • <strong>Cardiology</strong>/<strong>Endocrinology</strong>


congestive heart failure hospitalized for a decompensation episode.<br />

Eur Heart J 2009;30:2186-92.<br />

PubMed Link<br />

2. Rodgers JE, Lee, CR. Acute decompensated heart failure. In:<br />

DiPiro JT, Talbert RL, Yee G, et al., eds. Pharmacotherapy: A<br />

Pathophysiologic Approach, 8th ed. New York: McGraw-Hill,<br />

2011:173-216.<br />

6. Which one of the following interventions is likely to<br />

have the greatest clinical impact for K.T.?<br />

A. Education regarding weight monitoring.<br />

B. Dietary counseling.<br />

C. Follow-up telephone monitoring after discharge.<br />

D. Smoking cessation counseling.<br />

6. Answer: A<br />

Follow-up telephone monitoring, smoking cessation,<br />

and dietary counseling are all important interventions.<br />

However, only daily weight monitoring has been shown to<br />

decrease hospitalizations significantly and should always<br />

be emphasized in patient education (Answer A is correct;<br />

Answer B, Answer C, and Answer D are incorrect).<br />

1. Weintraub NL, Collins SP, Pang PS, et al. Acute heart<br />

failure syndromes: emergency department presentation, treatment,<br />

and disposition: current approaches and future aims.<br />

Circulation 2010;122:1975-96.<br />

PubMed Link<br />

2. Lindenfeld J, Albert NM, Boehmer JP, et al. Heart Failure<br />

Society of America 2010 comprehensive heart failure practice<br />

guidelines. J Card Fail 2010;16:e1-e194.<br />

PubMed Link<br />

C. Initiate clopidogrel 75 mg/day.<br />

D. Discontinue spironolactone.<br />

7. Answer: A<br />

It would be inappropriate to empirically discontinue any<br />

HF medication that has been associated with a decrease in<br />

mortality in patients with HF. There is no indication that<br />

this patient is intolerant of either hydralazine/nitrates or<br />

spironolactone (Answer B and Answer D are incorrect).<br />

Aspirin therapy is indicated in all patients receiving LVADs<br />

to reduce thromboembolic events and pump thrombosis<br />

(Answer A is correct). The Boyle, et al. study did show a<br />

low incidence of thromboembolic events in patients treated<br />

with aspirin and low-intensity warfarin. However, it is<br />

unknown whether the addition of clopidogrel would result<br />

in a further reduction of events (Answer C is incorrect).<br />

1. Slaughter MS, Pagani FD, Rogers JG, et al. Clinical management<br />

of continuous-flow left ventricular assist devices in advanced<br />

heart failure. J Heart Lung Transplant 2010;29:S1-S39.<br />

PubMed Link<br />

2. Boyle AJ, Russell SD, Teuteberg JJ, et al. Low thromboembolism<br />

and pump thrombosis with the HeartMate II left<br />

ventricular assist device: analysis of outpatient anticoagulation.<br />

J Heart Lung Transplant 2009;28:881-7.<br />

PubMed Link<br />

3. Uriel N, Pak SW, Jorde UP, et al. Acquired von willebrand<br />

syndrome after continuous-flow mechanical device support<br />

contributes to a high prevalence of bleeding during long-term<br />

support and at the time of transplantation. J Am Coll Cardiol<br />

2010;56:1207-13.<br />

PubMed Link<br />

Questions 7–9 pertain to the following case.<br />

M.M. is a 65-year-old African American man with a history<br />

of ischemic cardiomyopathy (EF 10%) who is admitted for<br />

left ventricular assist device (LVAD) evaluation. He reports<br />

that he is “always tired lately” and that he is no longer able<br />

to complete his activities of daily living without resting.<br />

Pertinent medical history includes atrial fibrillation, hypertension,<br />

and stroke. Current drugs include warfarin 3 mg/<br />

day, enalapril 10 mg twice daily, metoprolol succinate 100<br />

mg/day, spironolactone 25 mg/day, hydralazine 25 mg<br />

twice daily, and isosorbide dinitrate 20 mg twice daily.<br />

Vital signs include BP 100/70 mm Hg, heart rate 70 beats/<br />

minute, respiratory rate 20 breaths/minute, and oxygen saturation<br />

96% on 2 L nasal cannula. M.M. is approved for<br />

LVAD, and he is successfully implanted with a HeartMate<br />

II device.<br />

7. Which one of the following therapy changes would be<br />

best before M.M’s discharge?<br />

A. Initiate aspirin 81 mg/day.<br />

B. Discontinue hydralazine and isosorbide dinitrate.<br />

8. Which one of the following is the best choice for managing<br />

M.M’s warfarin therapy?<br />

A. Continue therapy for INR goal 1.5–2.5.<br />

B. Continue therapy with INR goal 2–3.<br />

C. Continue therapy with INR goal 2.5–3.5.<br />

D. Discontinue therapy.<br />

8. Answer: B<br />

All patients with LVADs should receive anticoagulation<br />

to prevent pump thrombosis and thromboembolic events<br />

(Answer D is incorrect). Early anticoagulation therapy with<br />

LVADs was quite aggressive, with higher INR goals; however,<br />

recent data indicate that a lower target range of 1.5-2.5<br />

results in a low incidence of thromboembolism and also<br />

minimizes bleeding. However, patients with a separate indication<br />

for anticoagulation – such as atrial fibrillation with a<br />

CHADS 2<br />

score of 4, as is the case with this patient – warrant<br />

traditional INR goals of 2–3 (Answer B is correct). An INR<br />

goal of 1.5–2.5 may not provide adequate stroke prevention<br />

in the face of concomitant atrial fibrillation (Answer A is<br />

incorrect). An INR goal of 2.5–3.5 could increase the risk<br />

of bleeding (Answer C is incorrect).<br />

PSAP 2013 • <strong>Cardiology</strong>/<strong>Endocrinology</strong> 3 Answers


1. Boyle AJ, Russell SD, Teuteberg JJ, et al. Low thromboembolism<br />

and pump thrombosis with the HeartMate II left<br />

ventricular assist device: analysis of outpatient anticoagulation.<br />

J Heart Lung Transplant 2009;28:881-7.<br />

PubMed Link<br />

2. John R, Kamdar F, Liao K, et al. Low thromboembolic risk for<br />

patients with the HeartMate II left ventricular assist device. J<br />

Thorac Cardiovasc Surg 2008;136:1318-23.<br />

PubMed Link<br />

3. Slaughter MS, Pagani FD, Rogers JG, et al. Clinical management<br />

of continuous-flow left ventricular assist devices in advanced<br />

heart failure. J Heart Lung Transplant 2010;29:S1-S39.<br />

PubMed Link<br />

9. Three months later, M.M. presents to the emergency<br />

department with dizziness, fever, chills, and general<br />

malaise. Which one of the following is best to recommend<br />

for M.M.?<br />

A. Begin antibiotic therapy targeted at gram-positive<br />

organisms.<br />

B. Begin broad-spectrum antibiotic therapy and<br />

admit for monitoring.<br />

C. Be closely monitored and initiate antibiotic<br />

therapy only if cultures are positive.<br />

D. Begin broad-spectrum antibiotic therapy and<br />

discharge to home.<br />

9. Answer: B<br />

Most LVAD infections are caused by gram-positive bacteria;<br />

however, gram-negative bacteria need to be considered<br />

(Answer A is incorrect). Because the consequences of infection<br />

can be devastating in this population, it is recommended<br />

that patients be admitted and treated with intravenous antibiotic<br />

therapy (Answer B is correct). Answer C and Answer<br />

D are incorrect in that they do not warrant the patient to be<br />

admitted to the hospital for close monitoring.<br />

1. Slaughter MS, Pagani FD, Rogers JG, et al. Clinical management<br />

of continuous-flow left ventricular assist devices in advanced<br />

heart failure. J Heart Lung Transplant 2010;29:S1-S39.<br />

PubMed Link<br />

2. Topkara VK, Kondareddy S, Malik F, et al. Infectious complications<br />

in patients with left ventricular assist device: etiology<br />

and outcomes in the continous-flow era. Ann Thorac Surg<br />

2010;90:1270-7.<br />

PubMed Link<br />

Questions 10–12 pertain to the following case.<br />

T.A. is 76-year-old man who presents to the hospital with<br />

an acute HF exacerbation. He experiences dyspnea on<br />

exertion, 4-pillow orthopnea, and a 15-lb weight gain over<br />

3 weeks. Physical examination reveals JVD to the angle<br />

of the jaw, positive S3 heart sound, bilateral rales on auscultation,<br />

and 3+ bilateral lower extremity edema to the<br />

thighs. Chest radiography reveals pulmonary edema and<br />

pleural effusions. T.A.’s medical history is significant for<br />

non–ST-elevation myocardial infarction with four-vessel<br />

coronary artery bypass grafting 15 years ago, type 2 diabetes<br />

mellitus, and permanent atrial fibrillation. Vital signs<br />

include BP 119/72 mm Hg, heart rate 82 beats/minute,<br />

respiratory rate 26 breaths/minute, and oxygen saturation<br />

92%. Electrocardiography reveals several episodes of<br />

nonsustained ventricular tachycardia in the emergency<br />

department. His LVEF 1 month ago was 25%. Laboratory<br />

values include sodium 128 mEq/L, potassium 4.1 mEq/L,<br />

chloride 100 mEq/L, carbon dioxide 25 mEq/L, BUN 34<br />

mg/dL (baseline 24 mg/dL), and SCr 1.3 mg/dL (baseline<br />

SCr 0.9 mg/dL). Drugs on admission include lisinopril 20<br />

mg/day, carvedilol 12.5 mg twice daily, bumetanide 1 mg<br />

twice daily, hydralazine 50 mg three times/day, isosorbide<br />

dinitrate 20 mg three times/day, aspirin 81 mg/day, warfarin<br />

5 mg/day, and insulin glargine 35 units at bedtime.<br />

10. Which one of the following is the best initial diuretic<br />

therapy for T.A.?<br />

A. Bumetanide 1 mg intravenously.<br />

B. Bumetanide 2 mg intravenously.<br />

C. Metolazone 10 mg orally.<br />

D. Nesiritide 0.02 mcg/kg/minute intravenous<br />

continuous infusion.<br />

10. Answer: B<br />

On the basis of the signs and symptoms described, this<br />

patient has acute HF with fluid overload. For patients on<br />

a loop diuretic at home, initial management should consist<br />

of doubling the home dose and intravenous administration<br />

(Answer B is correct). It is essential to consider the absorption<br />

properties of loop diuretics to determine appropriate<br />

conversion from oral to intravenous therapy. Bumetanide<br />

has approximately 90% bioavailability, so the oral dose<br />

must be doubled to provide twice the home dose (Answer<br />

A is incorrect). Metolazone could be considered, but the<br />

dose listed is excessive, and this agent may cause severe<br />

electrolyte abnormalities in a patient with episodes of nonsustained<br />

VT (Answer C is incorrect). Nesiritide could be<br />

considered for symptomatic improvement in addition to<br />

intravenous loop diuretic, but this agent provides no mortality<br />

benefits or reductions in length of stay (Answer D is<br />

incorrect).<br />

1. Lindenfeld J, Albert NM, Boehmer JP, et al. Heart Failure<br />

Society of America 2010 comprehensive heart failure practice<br />

guidelines. J Card Fail 2010;16:e1-e194.<br />

PubMed Link<br />

2. Felker GM. Diuretic management in heart failure. Congest<br />

Heart Fail 2010;16(suppl 1):S68-S72.<br />

PubMed Link<br />

Answers<br />

4<br />

PSAP 2013 • <strong>Cardiology</strong>/<strong>Endocrinology</strong>


11. Which one of the following would be best for rapidly<br />

improving T.A.’s symptoms?<br />

A. Tolvaptan 30 mg orally once daily.<br />

B. Chlorothiazide 500 mg intravenously once daily.<br />

C. Nitroprusside 4 mcg/kg/minute intravenous<br />

continuous infusion.<br />

D. Nitroglycerin 30 mcg/minute intravenous<br />

continuous infusion.<br />

11. Answer: D<br />

This patient is experiencing symptomatic ADHF characterized<br />

by dyspnea on exertion, orthopnea, low oxygen<br />

saturation, and tachypnea. Vasodilators are indicated for<br />

rapid symptomatic relief. The patient’s blood pressure will<br />

tolerate a vasodilator; therefore, nitroglycerin is the best<br />

choice (Answer D is correct). Nitroprusside is a reasonable<br />

option, but the dose of 4 mcg/kg/minute is an overly<br />

aggressive dose (Answer C is incorrect). This patient has not<br />

been proved to be diuretic resistant; therefore, appropriate<br />

loop diuretic dosing should produce a symptom response.<br />

The patient’s sodium is also low, and this can be further<br />

exacerbated by sequential nephron blockade (Answer B is<br />

incorrect). Tolvaptan has been shown to improve symptoms<br />

in ADHF but not as rapidly as intravenous vasodilators, and<br />

AVP antagonists are not recommended in guidelines. The<br />

description of this patient does not suggest symptomatic<br />

hyponatremia (Answer A is incorrect).<br />

1. Elkayam U, Tasissa G, Binanay C, et al. Use and impact of inotropes<br />

and vasodilator therapy in hospitalized patients with<br />

severe heart failure. Am Heart J 2007;153:98-104.<br />

PubMed Link<br />

2. Konstam MA, Gheorghiade M, Burnett JC, et al. Effects of oral<br />

tolvaptan in patients hospitalized for worsening heart failure:<br />

the EVEREST Outcome Trial. JAMA 2007;297:1319-31.<br />

PubMed Link<br />

3. Peacock WF, Fonarow GC, Emerman CL, et al. Impact of early<br />

initiation of intravenous therapy for acute decompensated heart<br />

failure on outcomes in ADHERE. <strong>Cardiology</strong> 2007;107:44-51.<br />

PubMed Link<br />

4. Peacock WF, Emerman C, Costanzo MR, et al. Early vasoactive<br />

drugs improve heart failure outcomes. Congest Heart Fail<br />

2009;15:256-64.<br />

PubMed Link<br />

5. Stough WG, O’Connor CM, Gheorghiade M. Overview of current<br />

noninodilator therapies for acute heart failure syndromes.<br />

Am J Cardiol 2005;96[suppl]:41G–6G).<br />

PubMed Link<br />

12. Three hours after initiation of therapy and monitoring,<br />

T.A. has produced 200 mL of urine. He says his dyspnea<br />

has not improved. His current vital signs include<br />

BP 110/70 mm Hg, heart rate 79 beats/minute,<br />

respiratory rate 20 breaths/minute, and oxygen saturation<br />

93%. Which one of the following would be best to<br />

recommend for T.A.?<br />

A. Increase to bumetanide 4 mg intravenously.<br />

B. Initiate ultrafiltration.<br />

C. Initiate milrinone 0.375 mcg/kg/minute.<br />

D. Continue the current regimen because symptoms<br />

have improved.<br />

12. Answer: A<br />

Patients without kidney dysfunction should respond by<br />

producing at least 500 mL of urine 2 hours after administration.<br />

If this end point is not achieved, clinicians should<br />

consider options for more aggressive volume removal.<br />

Because this patient was taking bumetanide 1 mg twice<br />

daily at home and was given bumetanide 2 mg intravenously<br />

initially, his dose should be increased to bumetanide<br />

4 mg intravenous at this time (Answer A is correct).<br />

Ultrafiltration is a reasonable option; however, this patient<br />

has not yet failed diuretics, so it is too soon for this costly<br />

intervention (Answer B is incorrect). Milrinone should not<br />

be added because the patient has an adequate blood pressure<br />

and is not showing signs of low CO. He also has runs of<br />

nonsustained ventricular tachycardia, which may be exacerbated<br />

by inotropes (Answer C is incorrect). Continuing the<br />

current regimen is inappropriate because this patient had<br />

several signs and symptoms of fluid overload on admission<br />

and did not respond adequately to the initial bumetanide<br />

dose (Answer D is incorrect).<br />

1. Abraham WT, Adams KF, Fonarow GC, et al. In-hospital<br />

mortality in patients with acute decompensated heart failure<br />

requiring intravenous vasoactive medications: an analysis<br />

from the acute decompensated heart failure national registry<br />

(ADHERE). J Am Coll Cardiol 2005;46:57-64.<br />

PubMed Link<br />

2. Cuffe MS, Califf RM, Adams KF, et al. Short-term intravenous<br />

milrinone for acute exacerbation of chronic heart failure: a randomized<br />

controlled trial. JAMA 2002;287:1541-7.<br />

PubMed Link<br />

3. DiDomenico RJ, Park HY, Southworth MR, et al. Guidelines<br />

for acute decompensated heart failure treatment. Ann<br />

Pharmacother 2004;38:649-60.<br />

PubMed Link<br />

4. Felker GM. Diuretic management in heart failure. Congest<br />

Heart Fail 2010;16(suppl 1):S68-S72.<br />

PubMed Link<br />

Questions 13 and 14 pertain to the following case.<br />

A.L. is a 73-year-old woman who presents to the emergency<br />

department with extreme dyspnea on exertion when<br />

walking from one room to the next at home. She has experienced<br />

lethargy and gradual reduction in exercise tolerance<br />

for the past 3 months. A.L. also admits feeling cold most of<br />

PSAP 2013 • <strong>Cardiology</strong>/<strong>Endocrinology</strong> 5 Answers


the day. She has been adherent to her dietary restrictions<br />

and drugs. Her weight has been stable at 85 kg during this<br />

time. She has a history of alcohol-induced nonischemic<br />

cardiomyopathy (LVEF 15%). Vital signs on admission<br />

include BP 82/56 mm Hg (without orthostasis), heart rate<br />

95 beats/minute, and respiratory rate 18 breaths/minute.<br />

On physical examination, she shows a stable 6-cm JVD,<br />

no pulmonary edema, no ascites, and no lower extremity<br />

edema. Laboratory analysis reveals sodium 135 mmol/L,<br />

potassium 4.1 mmol/L, carbon dioxide 28 mEq/L, chloride<br />

99 mEq/L, BUN 42 mg/dL (baseline 34 mg/dL),<br />

SCr 1.8 mg/dL (baseline 1.4 mg/dL), and magnesium 1.8<br />

mg/dL. A.L. has been stable on her oral HF drugs, which<br />

include metoprolol succinate 50 mg once daily, enalapril 5<br />

mg twice daily, and spironolactone 25 mg once daily.<br />

13. Which one of the following hemodynamic subsets best<br />

describes A.L.?<br />

A. Warm and dry.<br />

B. Warm and wet.<br />

C. Cold and dry.<br />

D. Cold and wet.<br />

13. Answer: C<br />

This patient shows signs and symptoms of low CO (i.e.,<br />

lethargy and reduced exercise tolerance) that have progressively<br />

worsened the past several months. Additionally, her<br />

vital signs (e.g., BP of 82/56 mm Hg) and laboratory profile<br />

(e.g., SCr = 1.8 mg/dL) are consistent with this low CO,<br />

suggesting that she is “cold.” Although the patient has been<br />

adherent to her diet and drug regimens, she has no symptoms<br />

of volume overload such as peripheral or pulmonary<br />

edema that would suggest that she is “wet.” The cold and dry<br />

hemodynamic subset best describes her current condition<br />

(Answer C is correct; Answer A, Answer B, and Answer D<br />

are incorrect).<br />

1. Hunt SA, Abraham WT, Chin MH, et al. 2009 focused update<br />

incorporated into the ACC/AHA 2005 Guidelines for the<br />

Diagnosis and Management of Heart Failure in Adults: a report<br />

of the American College of <strong>Cardiology</strong> Foundation/American<br />

Heart Association Task Force on Practice Guidelines: developed<br />

in collaboration with the International Society for Heart<br />

and Lung Transplantation. Circulation 2009;119:e391-e479.<br />

PubMed Link<br />

2. McMurray JV, Adamopoulos S, Anker SD, et al. European Society<br />

of <strong>Cardiology</strong> guidelines for the diagnosis and treatment of acute<br />

and chronic heart failure. Eur Heart J 2012;33:1787-847.<br />

PubMed Link<br />

14. Which one of the following agents would most likely<br />

benefit A.L. at this time?<br />

A. Milrinone 0.375 mcg/kg/minute.<br />

B. Dobutamine 5 mcg/kg/minute.<br />

C. Nitroglycerin 10 mcg/minute.<br />

D. Furosemide 40 mg intravenously.<br />

14. Answer: B<br />

This patient has progressive worsening of HF leading to<br />

acute decompensation characterized by a cold and dry<br />

profile with symptomatic hypotension; therefore, nitroglycerin<br />

would not be an appropriate option (Answer C is<br />

incorrect). Because she has no signs or symptoms of being<br />

“wet,” a loop diuretic would not be an appropriate choice<br />

because furosemide could deplete her intervascular volume<br />

(Answer D is incorrect). Her worsening kidney function,<br />

reduced exercise tolerance, and feeling of being cold is<br />

likely caused by the reduction in CO; thus she may warrant<br />

an inotrope. Because of its mechanism of action, milrinone<br />

has an increased risk of hypotension and its effects<br />

may be prolonged in patients with kidney dysfunction. This<br />

patient is already hypotensive and has kidney dysfunction;<br />

therefore, milrinone would not be an appropriate choice<br />

(Answer A is incorrect). Dobutamine would be the best<br />

choice to improve CO and limit the risk of drug-induced<br />

hypotension despite the patient being on a β-blocker at<br />

home (Answer B is correct).<br />

1. Cuffe MS, Califf RM, Adams KF, et al. Short-term intravenous<br />

milrinone for acute exacerbation of chronic heart failure: a randomized<br />

controlled trial. JAMA 2002;287:1541-7.<br />

PubMed Link<br />

2. Lindenfeld J, Albert NM, Boehmer JP, et al. Heart Failure<br />

Society of America 2010 comprehensive heart failure practice<br />

guidelines. J Card Fail 2010;16:e1-e194.<br />

PubMed Link<br />

Questions 15–18 pertain to the following case.<br />

C.J. is a 75-year-old man (height 5′8′′, weight 72 kg) with<br />

ischemic cardiomyopathy (EF 30%) presenting to the cardiovascular<br />

intensive care unit with acute decompensated<br />

heart failure (ADHF) and symptoms of worsening dyspnea<br />

on exertion, orthopnea, and paroxysmal nocturnal<br />

dyspnea. His medical history includes coronary artery<br />

disease, hyperlipidemia, hypertension, and chronic kidney<br />

disease. An implantable cardioverter-defibrillator was<br />

placed 2 years ago after an episode of ventricular tachycardia,<br />

but there have been no occurrences since then. His<br />

vital signs are BP 117/68 mm Hg, heart rate 75 beats/<br />

minute, respiratory rate 23 breaths/minute, and oxygen<br />

saturation 94% on 4 L nasal cannula. He is noted to have<br />

a JVD up to his ear, rales bilaterally, and 3+ bilateral lower<br />

extremity edema up to his knees, but only mild abdominal<br />

edema. He admits a 20-lb weight gain in the past 2<br />

weeks after running out of his home drugs. Pertinent laboratory<br />

values include sodium 136 mmol/L, potassium 4.8<br />

mmol/L, BUN 59 mg/dL, SCr 2.3 mg/dL (baseline 1.7<br />

mg/dL), N-terminal proBNP 9543 pg/mL, AST 28 U/L,<br />

Answers<br />

6<br />

PSAP 2013 • <strong>Cardiology</strong>/<strong>Endocrinology</strong>


ALT 32 U/L, and lactate dehydrogenase 125 U/L. C.J.’s<br />

drugs include atorvastatin 40 mg/day, aspirin 81 mg/day,<br />

enalapril 10 mg twice daily, carvedilol 12.5 mg twice daily,<br />

digoxin 125 mcg/day, and furosemide 40 mg twice daily.<br />

15. Based on current literature, which one of the following<br />

would be most effective for rapid onset volume removal<br />

and symptom relief without long-term adverse effects<br />

in C.J.?<br />

A. Furosemide 40 mg intravenously every 12 hours.<br />

B. Furosemide 100 mg intravenously every 12 hours.<br />

C. Furosemide 4 mg/hour intravenous continuous<br />

infusion.<br />

D. Furosemide 8 mg/hour intravenous continuous<br />

infusion.<br />

15. Answer: B<br />

Results from the Diuretics Optimization Strategies<br />

Evaluation (DOSE) trial, a large, randomized controlled<br />

study, have helped clinicians better understand the most<br />

effective intravenous loop diuretic dosing regimen in<br />

patients with ADHF. In this trial, high-dose loop diuretics<br />

produced more net fluid loss, weight loss, and relief from<br />

dyspnea with only temporary worsening of kidney function.<br />

On the basis of these data, aggressive diuresis with<br />

furosemide is appropriate for this patient (Answer B is correct)<br />

rather than conservative dosing to rapidly improve<br />

dyspnea symptoms (Answer A is incorrect). Continuous<br />

infusion was found to be no more effective than intravenous<br />

bolus dosing. Furthermore, an intravenous bolus<br />

should be administered before starting a continuous infusion<br />

to achieve more rapid diuresis and improvement in<br />

symptoms (Answer C and Answer D are incorrect).<br />

1. DiDomenico RJ, Park HY, Southworth MR, et al. Guidelines<br />

for acute decompensated heart failure treatment. Ann<br />

Pharmacother 2004;38:649-60.<br />

PubMed Link<br />

2. Peacock WF, Costanzo MR, De Marco T, et al. Impact of intravenous<br />

diuretics on the outcomes of patients hospitalized with<br />

acute decompensated heart failure: insights from the ADHERE<br />

registry. <strong>Cardiology</strong> 2009;113:12-9.<br />

PubMed Link<br />

3. Felker GM, Lee KL, Bull DA, et al. Diuretic strategies in<br />

patients with acute decompensated heart failure. N Engl J Med<br />

2011;364:797-805.<br />

PubMed Link<br />

16. C.J. responds poorly to the initial diuretic dosing, producing<br />

only 800 mL of urine overnight. Which one of<br />

the following would be best to recommend for overcoming<br />

possible diuretic resistance and successfully<br />

removing excess volume for C.J.?<br />

A. Metolazone 5 mg by mouth daily.<br />

B. Chlorothiazide 1000 mg intravenously twice<br />

daily.<br />

C. Hydrochlorothiazide 100 mg by mouth daily.<br />

D. Spironolactone 25 mg by mouth daily.<br />

16. Answer: A<br />

After escalating doses of intravenous loop diuretic fail to<br />

resolve symptoms, sequential nephron blockade is a reasonable<br />

option and is recommended in guidelines to overcome<br />

diuretic resistance. Because this patient has chronic kidney<br />

disease (baseline CrCl about 40 mL/minute) with worsening<br />

kidney function (current CrCl about 30 mL/minute),<br />

metolazone is a good option because it is less dependent<br />

on glomerular filtration to reach the site of action (Answer<br />

A is correct). The patient has only mild abdominal edema,<br />

so absorption should not be affected. Chlorothiazide and<br />

hydrochlorothiazide could be considered, but these may not<br />

reach the intended site of action because of dependence on<br />

adequate glomerular filtration. The chlorothiazide and hydrochlorothiazde<br />

dosing options are also high for a starting dose<br />

(Answer B and Answer C are incorrect). Spironolactone is<br />

not a potent diuretic and may be considered for chronic therapy;<br />

however, this drug is rarely used to augment diuresis in<br />

ADHF (Answer D is incorrect).<br />

1. Jentzer JC, DeWald TA, Hernandez AF. Combination loop<br />

diuretics with thiazide-type diuretics in heart failure. J Am Coll<br />

Cardiol 2010;56:1527-34.<br />

PubMed Link<br />

2. Lindenfeld J, Albert NM, Boehmer JP, et al. Heart Failure<br />

Society of America 2010 comprehensive heart failure practice<br />

guidelines. J Card Fail 2010;16:e1-e194.<br />

PubMed Link<br />

17. Because of C.J.’s poor initial response, a right heart catheterization<br />

is performed for hemodynamic assessment.<br />

Key results include central venous pressure 10 mm Hg,<br />

pulmonary artery systolic/diastolic pressure 44/30 mm<br />

Hg, pulmonary capillary wedge pressure (PCWP) 29<br />

mm Hg, CO 3.5 L/minute, cardiac index (CI) 1.9 L/<br />

minute/m 2 , systemic vascular resistance (SVR) 2000<br />

dynes/second/cm -5 , and systemic BP 105/65 mm Hg.<br />

Which one of the following would be best to recommend<br />

for hemodynamic improvement and to minimize<br />

the risk of in-hospital mortality for C.J.?<br />

A. Dobutamine 5 mcg/kg/minute.<br />

B. Dopamine 10 mcg/kg/minute.<br />

C. Nesiritide 2 mcg/kg bolus followed by 0.03 mcg/<br />

kg/minute.<br />

D. Nitroprusside 0.2 mcg/kg/minute.<br />

17. Answer: D<br />

This patient is characterized into the cold and wet subset<br />

on the basis of symptoms and now hemodynamic evidence<br />

PSAP 2013 • <strong>Cardiology</strong>/<strong>Endocrinology</strong> 7 Answers


from right heart catheterization. Inotropes have been<br />

linked to increased in-hospital mortality and ventricular<br />

arrhythmias; therefore, they should be used only when<br />

necessary if patients show symptoms of low CO and hypotension<br />

or for progression to end-stage HF (Answer A and<br />

Answer B are incorrect). Vasodilators are the preferred<br />

agents for this patient on the basis of his high PCWP (preload),<br />

high SVR (afterload), and low CI. Reducing preload<br />

and afterload to normal values reduces myocardial wall<br />

stress and optimizes CI. Nesiritide would be an appropriate<br />

drug selection, but the dosing here is too aggressive<br />

and may induce hypotension (Answer C is incorrect).<br />

Nitroprusside is a balanced arterial-venous dilator with<br />

fast onset and a short half-life. This patient has normal<br />

liver function and chronic kidney disease, but temporary<br />

initiation of nitroprusside at low doses is safe and effective,<br />

making nitroprusside the best choice in this case<br />

(Answer D is correct).<br />

1. Abraham WT, Adams KF, Fonarow GC, et al. In-hospital<br />

mortality in patients with acute decompensated heart failure<br />

requiring intravenous vasoactive medications: an analysis<br />

from the acute decompensated heart failure national registry<br />

(ADHERE). J Am Coll Cardiol 2005;46:57-64.<br />

PubMed Link<br />

2. Burger AJ, Horton DP, LeJemtel T, et al. Effect of nesiritide<br />

(B-type natriuretic peptide) and dobutamine on ventricular<br />

arrhythmias in the treatment of patients with acutely decompensated<br />

congestive heart failure: The PRECEDENT study.<br />

Am Heart J 2002;144:1102-8.<br />

PubMed Link<br />

3. Lindenfeld J, Albert NM, Boehmer JP, et al. Heart Failure<br />

Society of America 2010 comprehensive heart failure practice<br />

guidelines. J Card Fail 2010;16:e1-e194.<br />

PubMed Link<br />

4. Mullens W, Abrahams Z, Francis GS, et al. Sodium nitroprusside<br />

for advanced low-output heart failure. J Am Coll<br />

Cardiol 2008;52:200-7.<br />

PubMed Link<br />

C. Give conivaptan 20 mg intravenous bolus,<br />

followed by 20 mg administered intravenously<br />

over 24 hours.<br />

D. Give tolvaptan 30 mg by mouth daily.<br />

18. Answer: B<br />

The only guideline-recommended treatment for asymptomatic<br />

hyponatremia is fluid restriction. This patient<br />

has a rather liberal fluid restriction at this point, so further<br />

restriction is the best option (Answer B is correct).<br />

Diuretics can deplete electrolytes because the main mechanism<br />

for fluid removal is driven by the prevention of<br />

sodium reabsorption in renal tubules (Answer A is incorrect).<br />

Conivaptan has not been proved to be effective in<br />

patients with HF (Answer C is incorrect). Tolvaptan<br />

has FDA label approval for symptomatic hypervolemic<br />

or euvolemic hyponatremia or for sodium less than 125<br />

mEq/L. It is also effective for rapid symptom relief for<br />

patients with ADHF but provides no morbidity or mortality<br />

benefits (Answer D is incorrect).<br />

1. Gheorghiade M, Konstam MA, Burnett JC, et al. Shortterm<br />

clinical effects of tolvaptan, an oral vasopressin<br />

antagonist, in patients hospitalized for heart failure. JAMA<br />

2007;297:1332-43.<br />

PubMed Link<br />

2. Gheorghiade M, Gattis WA, O’Connor CM, et al. Effects of<br />

tolvaptan, a vasopressin antagonist, in patients hospitalized<br />

with worsening heart failure. JAMA 2004;291:1963-71.<br />

PubMed Link<br />

3. Konstam MA, Gheorghiade M, Burnett JC, et al. Effects of oral<br />

tolvaptan in patients hospitalized for worsening heart failure:<br />

the EVEREST Outcome Trial. JAMA 2007;297:1319-31.<br />

PubMed Link<br />

4. Lindenfeld J, Albert NM, Boehmer JP, et al. Heart Failure<br />

Society of America 2010 comprehensive heart failure practice<br />

guidelines. J Card Fail 2010;16:e1-e194.<br />

PubMed Link<br />

18. A 69-year-old man with a history of nonischemic cardiomyopathy<br />

is admitted to the hospital with severe<br />

volume overload after running out of his home loop<br />

diuretic 3 weeks ago. On admission, he was placed<br />

on a 2-L/day fluid restriction and given intravenous<br />

furosemide with successful removal of volume. He is<br />

almost back to his baseline weight but is now hyponatremic<br />

with a sodium concentration of 124 mmol/L.<br />

His mental status is at baseline, and he has no nausea,<br />

vomiting, or headache. Which one of the following is<br />

best to recommend for this patient’s hyponatremia?<br />

A. Continue the current plan because further<br />

volume removal will correct his sodium.<br />

B. Change his fluid restriction to 1.5 L/day and<br />

continue to monitor.<br />

19. A 65-year-old man admitted for progressive HF<br />

caused by ischemic cardiomyopathy has become<br />

refractory to most evidence-based medications. He<br />

has been hospitalized for ADHF six times in the<br />

past 4 months. Similar to his previous admissions,<br />

the patient presents with cold and wet symptoms.<br />

His evaluation for heart transplantation was completed<br />

during previous admissions. He also has a<br />

medical history of coronary artery disease, diabetes<br />

mellitus, hypertension, chronic kidney insufficiency,<br />

and hypothyroidism. At this admission, the patient<br />

has diminished urine output to escalating doses of<br />

a bumetanide intravenous continuous infusion and<br />

chlorothiazide intravenous bolus every 12 hours. His<br />

cardiologist would like to perform a right heart catheterization<br />

to optimize hemodynamics and place<br />

Answers<br />

8<br />

PSAP 2013 • <strong>Cardiology</strong>/<strong>Endocrinology</strong>


an intra-aortic balloon pump to move him up on the<br />

transplant list. The patient’s current laboratory values<br />

include sodium 131 mEq/L, potassium 3.9 mEq/L,<br />

carbon dioxide 28 mEq/L, chloride 101 mEq/L,<br />

BUN 55 mg/dL, SCr 2.2 mg/dL, magnesium 2.2 mg/<br />

dL, AST 205 U/L, ALT 191 U/L, and total bilirubin<br />

1.9 mg/dL. The right heart catheterization shows the<br />

following: central venous pressure 9 mm Hg, pulmonary<br />

artery systolic/diastolic pressure 52/25 mm Hg,<br />

PCWP 24 mm Hg, CO 3.1 L/minute, CI 1.6 L/minute/m<br />

2 , SVR 2040 dynes/second/cm -5 , and systemic<br />

BP 89/59 mm Hg. With this new information and his<br />

current clinical status, which one of the following is the<br />

best vasoactive drug to aid as a bridge to transplant in<br />

this patient?<br />

A. Milrinone 0.2 mcg/kg/minute.<br />

B. Dobutamine 10 mcg/kg/ minute.<br />

C. Nitroglycerin 10 mcg/minute.<br />

D. Nitroprusside 6 mcg/kg/ minute.<br />

19. Answer: A<br />

This patient was admitted to elevate his status on the transplant<br />

list. After completion of a right heart catheterization<br />

for hemodynamic evaluation, he was found to have a high<br />

preload and afterload with low CI. Milrinone has multiple<br />

hemodynamic properties, including preload and afterload<br />

reduction plus augmentation of CO, which makes it a good<br />

choice for this patient (Answer A is correct). Dobutamine<br />

also increases CO but would have less effect on PCWP and<br />

SVR. Dobutamine 10 mcg/kg/minute is also too aggressive<br />

for a starting dose (Answer B is incorrect). Vasodilators<br />

are often used as a BTT to minimize myocardial workload.<br />

Nitroglycerin is good for short-term improvement in symptoms<br />

and hemodynamics, but it is limited by tachyphylaxis<br />

when used for prolonged periods (Answer C is incorrect).<br />

Nitroprusside is not the best choice in patients who may be<br />

on the transplant list for extended periods and may rapidly<br />

decompensate, leading to liver and renal insufficiency. Such<br />

organ dyfuncations can lead to potential cyanide toxicity.<br />

Additionally, the starting dose of nitroprusside ( 6 mcg/kg/<br />

minute) is too aggressive (Answer D is incorrect).<br />

1. Hunt SA, Abraham WT, Chin MH, et al. 2009 focused update<br />

incorporated into the ACC/AHA 2005 Guidelines for the<br />

Diagnosis and Management of Heart Failure in Adults: a report<br />

of the American College of <strong>Cardiology</strong> Foundation/American<br />

Heart Association Task Force on Practice Guidelines: developed<br />

in collaboration with the International Society for Heart<br />

and Lung Transplantation. Circulation 2009;119:e391-e479.<br />

PubMed Link<br />

2. Lindenfeld J, Albert NM, Boehmer JP, et al. Heart Failure<br />

Society of America 2010 comprehensive heart failure practice<br />

guidelines. J Card Fail 2010;16:e1-e194.<br />

PubMed Link<br />

20. The cardiology section at your hospital wants to add<br />

nesiritide as part of the routine standing order set for<br />

all patients admitted with ADHF, volume overload,<br />

and systolic BP greater than 90 mm Hg. Given your<br />

knowledge as a pharmacist reviewer, which one of the<br />

following provides the most accurate feedback on the<br />

safety and efficacy of nesiritide?<br />

A. Disagree; there are clear data showing increases<br />

in mortality and kidney dysfunction when used<br />

routinely.<br />

B. Disagree; there are no data showing a decrease<br />

in morbidity, mortality, or hospital readmissions<br />

to support routine use.<br />

C. Agree; the aggregate data support nesiritide use<br />

to decrease length of stay and prevent hospital<br />

readmissions.<br />

D. Agree; nesiritide has the benefit of lowering<br />

loop diuretic requirements, which translates to<br />

decreases in overall mortality.<br />

20. Answer: B<br />

At this time, there are no data to support the routine use of<br />

nesiritide; it has shown hemodynamic benefits comparable<br />

with nitroglycerin, but these data have been criticized<br />

for nitroglycerin underdosing. The ASCEND-HF study<br />

was not able to show a significant difference in dyspnea<br />

score or the combined end point of hospital readmissions<br />

or death (Answer B is correct). Meta-analyses have also<br />

shown a possible link to kidney dysfunction and mortality<br />

with nesiritide; however, these analyses have been<br />

criticized because of the studies selected, and the results<br />

are hypothesis generating at best (Answer A is incorrect).<br />

Nesiritide does not decrease LOS, prevent hospital readmissions,<br />

or lower diuretic requirements (Answer C and<br />

Answer D are incorrect.<br />

1. O’Connor CM, Hernandez AF, Armstrong PW, et al. Effect<br />

of nesiritide in patients with acute decompensated heart failure.<br />

N Engl J Med 2011;365:32-43.<br />

PubMed Link<br />

2. Sackner-Bernstein JD, Skopicki HA, Aaronson KD. Risk<br />

of worsening renal function with nesiritide in patients<br />

with acutely decompensated heart failure. Circulation<br />

2005;111:1487-91.<br />

PubMed Link<br />

3. Sackner-Bernstein JD, Kowalski M, Fox M, et al. Short-term<br />

risk of death after treatment with nesiritide for decompensated<br />

heart failure. JAMA 2005;293:1900-5.<br />

PubMed Link<br />

4. VAMC Investigators. Intravenous nesiritide versus nitroglycerin<br />

for treatment of decompensated congestive heart failure.<br />

JAMA 2002;287:1531-40.<br />

PubMed Link<br />

PSAP 2013 • <strong>Cardiology</strong>/<strong>Endocrinology</strong> 9 Answers


Newer Antithrombotic Agents<br />

and Their Role in ACS<br />

21. A 60-year-old woman (weight 85 kg) presents to the<br />

emergency department with crushing substernal chest<br />

pain and ST-segment elevations on electrocardiography.<br />

She has no significant medical history; she is prescribed<br />

a heparin intravenous infusion and aspirin 325 mg<br />

orally daily. Laboratory results are hemoglobin 12.0 g/<br />

dL, hematocrit 36%, and SCr 1.0 mg/dL. Her physician<br />

wants the fastest-acting P2Y12 antagonist regimen<br />

before percutaneous coronary intervention (PCI) and<br />

asks you for a recommendation. Which one of the following<br />

regimens is best to recommend for this patient?<br />

A. Clopidogrel 300 mg loading dose (LD), followed<br />

by 75 mg/day.<br />

B. Prasugrel 60 mg LD, followed by 10 mg/day.<br />

C. Clopidogrel 600 mg LD, followed by 75 mg/day.<br />

D. Ticagrelor 180 mg LD, followed by 90 mg twice<br />

daily.<br />

21. Answer: B<br />

Both ticagrelor and prasugrel have shown quicker onset of<br />

action than clopidogrel (Answer A and Answer C are incorrect).<br />

Answer D is incorrect because the onset for ticagrelor<br />

is 90 minutes compared with 30 minutes for prasugrel and<br />

the aspirin dose prescribed. Prasugrel’s onset of action is<br />

30 minutes, making it the fastest-acting P2Y12 antagonist<br />

(Answer B is correct).<br />

1. Jakubowski JA, Matsushima N, Asai F, et al. A multiple dose<br />

study of prasugrel (CS-747), a novel thienopyridine P2Y12<br />

inhibitor, compared with clopidogrel in healthy humans.Br J<br />

Clin Pharmacol 2007;63:421-30.<br />

PubMed Link<br />

2. Gurbel PA, Bliden KP, Butler K, et al. Randomized double-blind<br />

assessment of the ONSET and OFFSET of the<br />

antiplatelet effects of ticagrelor versus clopidogrel in patients<br />

with stable coronary artery disease: the ONSET/OFFSET<br />

study. Circulation 2009;120:2577-85.<br />

PubMed Link<br />

22. A 76-year-old man (weight 65 kg) presents to the<br />

emergency department with substernal chest pain,<br />

ST-segment depressions on electrocardiography, and<br />

elevated troponins. His medical history is significant for<br />

hypertension and a previous ischemic stroke. Laboratory<br />

results are hemoglobin 12.4 g/dL, hematocrit 37%, SCr<br />

1.2 mg/dL, and troponin 5.0 ng/mL. He is prescribed<br />

a heparin intravenous infusion and aspirin 81 mg orally<br />

daily with the plan for PCI. From the available evidence<br />

and the patient’s history, which one of the following is<br />

most appropriate to initiate for this patient?<br />

A. Clopidogrel 300 mg LD, followed by 75 mg/day.<br />

B. Prasugrel 60 mg LD, followed by 5 mg/day.<br />

C. Clopidogrel 600 mg LD, followed by 75 mg/day.<br />

D. Ticagrelor 180 mg LD, followed by 90 mg twice<br />

daily.<br />

22. Answer: D<br />

The patient is experiencing a non-ST segment elevation<br />

myocardial infarction with a planned PCI. Answer A and<br />

Answer C are incorrect because both ticagrelor and prasugrel<br />

have been proved superior to clopidogrel. The patient<br />

is 76 years old and had a previous stroke. The subgroup<br />

older than 75 years in the TRITON trial exhibited no difference<br />

in the net clinical benefit of prasugrel compared<br />

with clopidogrel. Additionally, the patients with a previous<br />

TIA/stroke had worse outcomes compared with clopidogrel<br />

(Answer B is incorrect). Also in the PLATO trial, the<br />

efficacy of ticagrelor over clopidogrel was not attenuated by<br />

age of greater than 75 years or history of ischemic stroke.<br />

Therefore, Answer D is the best choice.<br />

1. Wiviott SD, Braunwald E, McCabe CH, et al. Prasugrel versus<br />

clopidogrel in patients with acute coronary syndromes. N Engl<br />

J Med 2007;357:2001-15.<br />

PubMed Link<br />

2. Wallentin L, Becker RC, Budaj A, et al. Ticagrelor versus clopidogrel<br />

in patients with acute coronary syndromes. N Engl J<br />

Med 2009; 361:1045-57.<br />

PubMed Link<br />

23. A 70-year-old woman (weight 55 kg) presents to the<br />

emergency department with chest pain that radiates<br />

to her jaw and ST-segment depressions on electrocardiography.<br />

Her medical history is significant for<br />

diabetes mellitus, hypertension, and chronic kidney<br />

insufficiency. Laboratory results are hemoglobin 10.5<br />

g/dL, hematocrit 32%, SCr 1.8 mg/dL, and troponin<br />

less than 0.2 ng/mL. She is prescribed a heparin intravenous<br />

infusion and aspirin 325 mg orally daily. The<br />

medical team has scheduled the patient for a cardiac<br />

catheterization. From the available evidence and the<br />

patient’s history, which one of the following is the most<br />

appropriate antiplatelet regimen to initiate for this<br />

patient?<br />

A. Clopidogrel 300 mg LD, followed by 75 mg/day.<br />

B. Prasugrel 60 mg LD, followed by 5 mg/day.<br />

C. Clopidogrel 600 mg LD, followed by 75 mg/day.<br />

D. Ticagrelor 180 mg LD, followed by 90 mg twice<br />

daily.<br />

23. Answer: C<br />

This patient is experiencing unstable angina, is taking aspirin<br />

325 mg orally daily, weighs 55 kg, and is scheduled<br />

for cardiac catheterization. Answer B is incorrect because<br />

Answers<br />

10<br />

PSAP 2013 • <strong>Cardiology</strong>/<strong>Endocrinology</strong>


the patient’s weight is 55 kg. In the subgroups from the<br />

TRITON trial, prasugrel did not show a net clinical benefit<br />

over clopidogrel in patients who weighed less than 60<br />

kg. Answer D is incorrect because the patient is taking aspirin<br />

325 mg orally daily. Aspirin doses of 300 mg or greater<br />

predicted that clopidogrel would perform better than<br />

ticagrelor. The 600-mg loading dose of clopidogrel has been<br />

shown to be superior to 300 mg in this patient population<br />

(Answer C is correct; Answer A is incorrect).<br />

1. Wiviott SD, Braunwald E, McCabe CH, et al. Prasugrel versus<br />

clopidogrel in patients with acute coronary syndromes. N Engl<br />

J Med 2007;357:2001-15.<br />

PubMed Link<br />

2. Wallentin L, Becker RC, Budaj A, et al. Ticagrelor versus clopidogrel<br />

in patients with acute coronary syndromes. N Engl J<br />

Med 2009; 361:1045-57.<br />

PubMed Link<br />

3. Angiolillo DJ, Fernandez-Ortiz A, Bernardo E, et al. Variability<br />

in individual responsiveness to clopidogrel: clinical implications,<br />

management, and future perspectives. J Am Coll Cardiol<br />

2007;49:1505-16.<br />

PubMed Link<br />

24. A 65-year-old man (weight 72 kg) presents to the<br />

emergency department with chest pain, ST-segment<br />

depressions on electrocardiography, and elevated troponins.<br />

The patient’s medical history includes diabetes<br />

mellitus, hypertension, kidney transplantation, and<br />

previous Aspergillus pneumonia. The patient’s oral<br />

drugs are tacrolimus 2 mg twice daily, mycophenolate<br />

500 mg twice daily, voriconazole 200 mg twice daily,<br />

amlodipine 10 mg/day, aspirin 81 mg/day, pravastatin<br />

80 mg every evening, and enalapril 10 mg/day.<br />

Laboratory results are as follows: hemoglobin 11.5 g/<br />

dL, hematocrit 34%, and SCr 1.0 mg/dL. From the<br />

available evidence and the patient’s history, which one<br />

of the following is the most appropriate antiplatelet<br />

regimen to initiate in this patient?<br />

A. Clopidogrel 300 mg LD, followed by 75 mg/day.<br />

B. Prasugrel 60 mg LD, followed by 10 mg/day.<br />

C. Clopidogrel 600 mg LD, followed by 75 mg/day.<br />

D. Ticagrelor 180 mg LD, followed by 90 mg twice<br />

daily.<br />

24. Answer: B<br />

This patient presents with a non-ST segment myocardial<br />

infarction. He is recieving voriconazole for previous<br />

aspergillus pneumonia and has a history of kidney transplantation<br />

on tacrolimus. Answer A, Answer C, and Answer<br />

D are incorrect because significant CYP 3A4 interactions<br />

can occur with both clopidogrel and ticagrelor. Ticagrelor<br />

should be avoided in patients prescribed strong CYP 3A4<br />

inhibitors (e.g., voriconazole). Answer B is correct because<br />

there are no substantial drug interactions with strong CYP<br />

3A4 inhibitors and prasugrel.<br />

1. Bates ER, Lau WC, Angiolillo DJ. Clopidogrel-drug interactions.<br />

J Am Coll Cardiol 2011;57:1251-63.<br />

PubMed Link<br />

2. Farid NA, Payne CD, Small DS, et al. Cytochrome P450 3A<br />

inhibition by ketoconazole affects prasugrel and clopidogrel<br />

pharmacokinetics and pharmacodynamics differently. Clin<br />

Pharmacol Ther 2007;81:735-41.<br />

PubMed Link<br />

25. A 55-year-old man (weight 89 kg) presents to the<br />

emergency department with chest pain, ST-segment<br />

depressions on electrocardiography, and elevated troponins.<br />

His medical history includes asthma, diabetes<br />

mellitus, and hypertension. His drugs are amlodipine<br />

10 mg orally daily, aspirin 325 mg orally daily, atorvastatin<br />

80 mg orally every evening, and enalapril 10 mg<br />

orally daily. Laboratory results are hemoglobin 10.5 g/<br />

dL, hematocrit 32%, and SCr 1.0 mg/dL. Which one of<br />

the following would most affect this patient’s long-term<br />

outcomes if the medical team chose to use ticagrelor as<br />

the P2Y12 antagonist?<br />

A. Change the dose of aspirin to 81 mg orally daily.<br />

B. Educate the patient that his or her pulmonary<br />

function tests may decrease over time.<br />

C. Change ticagrelor to prasugrel because it is contraindicated<br />

in patients with asthma.<br />

D. Decrease the maintenance dose (MD) of ticagrelor<br />

to 45 mg orally twice daily because of the<br />

patient’s weight.<br />

25. Answer: A<br />

Answer B and Answer C are incorrect because ticagrelor is<br />

not contraindicated in asthma and does not effect pulmonary<br />

function tests. There is a significant increase in dyspnea<br />

initially with ticagrelor compared with clopidogrel, but this<br />

is not a contraindication for therapy. The patient should be<br />

counseled on this potential side effect. Answer D is incorrect<br />

because there is no weight cutoff for ticagrelor and no<br />

clinical outcome trial has studied ticagrelor at the 45 mg<br />

twice daily dosage. Answer A is correct in that those treated<br />

with aspirin doses less than 100 mg had better clinical outcomes<br />

with ticagrelor compared with clopidogrel. This has<br />

led to the recommendation that the maintenance dose of<br />

aspirin should be less than 100 mg when co-administered<br />

with ticagrelor.<br />

1. Storey RF, Becker RC, Harrington RA, et al. Pulmonary function<br />

in patients with acute coronary syndrome treated with<br />

ticagrelor or clopidogrel (from the Platelet Inhibition and<br />

Patient Outcomes [PLATO] pulmonary function substudy).<br />

Am J Cardiol 2011;108:1542-6.<br />

PubMed Link<br />

PSAP 2013 • <strong>Cardiology</strong>/<strong>Endocrinology</strong> 11 Answers


2. Wallentin L, Becker RC, Budaj A, et al. Ticagrelor versus clopidogrel<br />

in patients with acute coronary syndromes. N Engl J<br />

Med 2009; 361:1045-57.<br />

PubMed Link<br />

26. Which one of the following agents has the highest risk<br />

of fatal bleeding related to the competitor in acute coronary<br />

syndrome (ACS) in patients receiving PCI?<br />

A. Prasugrel 10 mg versus clopidogrel 75 mg/day.<br />

B. Rivaroxaban 2.5 mg twice daily versus placebo.<br />

C. Rivaroxaban 5 mg twice daily versus placebo.<br />

D. Vorapaxar 40 mg load; then 2.5 mg/day versus<br />

placebo.<br />

26. Answer: A<br />

Answer B, Answer C, and Answer D are incorrect because<br />

in their respective phase 3 clinical trials, rivaroxaban and<br />

vrapaxar did not increase the risk of fatal bleeding related<br />

to the comparator group. Answer A is correct based on data<br />

from the landmark TRITON trial. In this study, prasugrel<br />

significantly increased the risk of fatal bleeding compared<br />

with clopidogrel.<br />

1. Mega JL, Braunwald E, Wiviott SD, et al. Rivaroxaban in<br />

patients with a recent acute coronary syndrome. N Engl J Med<br />

2012;366:9-19.<br />

PubMed Link<br />

2. Tricoci P, Huang Z, Held C, et al. Thrombin-receptor antagonist<br />

vorapaxar in acute coronary syndromes. N Engl J Med<br />

2012;366:20-33.<br />

PubMed Link<br />

3. Wiviott SD, Braunwald E, McCabe CH, et al. Prasugrel versus<br />

clopidogrel in patients with acute coronary syndromes. N Engl<br />

J Med 2007;357:2001-15.<br />

PubMed Link<br />

27. Which one of the following best represents rivaroxaban’s<br />

potential role in the management of ACS given<br />

the available evidence?<br />

A. Early administration for a patient presenting with<br />

chest discomfort and ST-segment changes as a<br />

replacement for unfractionated heparin and lowmolecular-weight<br />

heparin.<br />

B. Administration during PCI to prevent stent thrombosis<br />

as a replacement for bivalirudin.<br />

C. Initiation just before hospital discharge in addition<br />

to aspirin and a thienopyridine for secondary prevention<br />

of ischemic events.<br />

D. A replacement for warfarin in patients with atrial<br />

fibrillation (AF).<br />

27. Answer: C<br />

The role of rivaroxaban in addition to aspirin or aspirin<br />

plus a thienopyridine for the secondary prevention of ischemic<br />

events is based on the ATLAS ACS 2 trial. Answer<br />

A is incorrect because it does not replace the early anticoagulant<br />

therapies in ACS. Answer B is incorrect because<br />

rivaroxaban has not been studied in PCI as a replacement<br />

for bivalirudin. Although it has label approval for atrial<br />

fibrillation, rivaroxaban dosing in atrial fibrillation is significantly<br />

different than the dosing in ACS; therefore, it should<br />

not be used in that fashion (Answer D is incorrect). Answer<br />

C is correct based on the inclusion criteria in the ATLAS<br />

ACS 2 trial.<br />

1.Mega JL, Braunwald E, Wiviott SD, et al. Rivaroxaban in<br />

patients with a recent acute coronary sndrome. N Engl J Med<br />

2012; 366:9-19.<br />

PubMed Link<br />

28. A 55-year-old man with ACS receives a stent to the left<br />

anterior descending artery. Which one of the following<br />

is most important to consider in choosing between<br />

ticagrelor and prasugrel for this patient?<br />

A. Efficacy compared with clopidogrel.<br />

B. Relative platelet inhibition compared with<br />

clopidogrel.<br />

C. Genetic effects on the drugs’ metabolism.<br />

D. Offset of the antiplatelet effects.<br />

28. Answer: D<br />

Answer A is incorrect because both ticagrelor and prasugrel<br />

have shown superiority over clopidogrel in randomized<br />

trials, and the populations in these studies were different.<br />

Based on these data, direct comparisons cannot be made.<br />

Ticagrelor and prasugrel both have superior platelet inhibition<br />

compared with clopidogrel (Answer B is incorrect).<br />

Answer C is incorrect because, to date, there have been no<br />

genetic effects identified that alter either drug’s metabolism.<br />

Answer D is correct based on Table 2-4, which compares<br />

the pharmacokinetic and pharmacodynamic parameters of<br />

clopidogrel, prasugrel, and ticagrelor.<br />

1. Wallentin L, Becker RC, Budaj A, et al. Ticagrelor versus clopidogrel<br />

in patients with acute coronary syndromes. N Engl J<br />

Med 2009;361:1045-57.<br />

PubMed Link<br />

2. Wiviott SD, Braunwald E, McCabe CH, et al. Prasugrel versus<br />

clopidogrel in patients with acute coronary syndromes. N Engl<br />

J Med 2007;357:2001-15.<br />

PubMed Link<br />

3. Gurbel PA, Bliden KP, Butler K, et al. Randomized double-blind<br />

assessment of the ONSET and OFFSET of the<br />

antiplatelet effects of ticagrelor versus clopidogrel in patients<br />

with stable coronary artery disease: the ONSET/OFFSET<br />

study. Circulation. 2009;120:2577-85.<br />

PubMed Link<br />

Answers<br />

12<br />

PSAP 2013 • <strong>Cardiology</strong>/<strong>Endocrinology</strong>


4. Jakubowski JA, Matsushima N, Asai F, et al. A multiple dose<br />

study of prasugrel (CS-747), a novel thienopyridine P2Y12<br />

inhibitor, compared with clopidogrel in healthy humans. Br J<br />

Clin Pharmacol 2007;63:421-30.<br />

PubMed Link<br />

29. Which one of the following best describes the potential<br />

role for cangrelor in ACS?<br />

A. As replacement of clopidogrel during PCI.<br />

B. An addition to clopidogrel during PCI to replace<br />

glycoprotein IIb/IIIa inhibitors.<br />

C. As therapy instead of an oral P2Y12 antagonist in<br />

patients awaiting coronary artery bypass grafting<br />

(CABG).<br />

D. As replacement for glycoprotein IIb/IIIa inhibitors<br />

in the medical management of ACS.<br />

29. Answer: C<br />

Based on the CHAMPION PCI and CHAMPION<br />

PLATFORM clinical trials, cangrelor has not been shown<br />

to be beneficial in replacing clopidogrel or when added to<br />

clopidogrel in PCI (Answer A and Answer B are incorrect).<br />

Answer D is incorrect because cangrelor has not been studied<br />

in this patient population. Answer C is correct based<br />

on the BRIDGE trial, which showed that cangrelor could<br />

be used instead of a P2Y12 antagonist in patients awaiting<br />

CABG.<br />

1. Bhatt DL, Lincoff AM, Gibson CM, et al. Intravenous platelet<br />

blockade with cangrelor during PCI. N Engl J Med<br />

2009;361:2330-41.<br />

PubMed Link<br />

2. Harrington RA, Stone GW, McNulty S, et al. Platelet inhibition<br />

with cangrelor in patients undergoing PCI. N Engl J Med<br />

2009; 361:2318-29.<br />

PubMed Link<br />

3. Angiolillo DJ, Firstenberg MS, Price MJ, et al. Bridging antiplatelet<br />

therapy with cangrelor in patients undergoing cardiac<br />

surgery. JAMA 2012;307:265-74.<br />

PubMed Link<br />

30. Which one of the following best describes the metabolic<br />

pathways of clopidogrel and ticagrelor?<br />

A. Both ticagrelor and clopidogrel are metabolized<br />

through CYP2D6 .<br />

B. Ticagrelor is metabolized through CYP2C9 but<br />

not clopidogrel.<br />

C. Both clopidogrel and ticagrelor are metabolized<br />

through CYP3A4.<br />

D. Ticagrelor is metabolized through CYP2C19 but<br />

not clopidogrel.<br />

30. Answer: C<br />

Clopidogrel is metabolized by CYP3A4, 2B6, 1A2, 2C9,<br />

and 2C19; ticagrelor is metabolized by CYP 3A4. Therefore,<br />

Answer A, Answer B, and Answer D are incorrect. Answer<br />

C is correct in that both clopidogrel and ticagrelor are<br />

metabolized through CYP3A4.<br />

1. Bates ER, Lau WC, Angiolillo DJ. Clopidogrel-drug interactions.<br />

J Am Coll Cardiol 2011;57:1251-63.<br />

PubMed Link<br />

2. Teng R, Oliver S, Hayes MA, et al. Absorption, distribution,<br />

metabolism, and excretion of ticagrelor in healthy subjects.<br />

Drug Metab Dispos 2010;38:1514-21.<br />

PubMed Link<br />

31. A 65-year-old man (weight 90 kg) was admitted for non–<br />

ST-segment elevation myocardial infarction (NSTEMI)<br />

and treated with a drug-eluting stent (DES). He received<br />

oral aspirin 325 mg and clopidogrel 600 mg at the time<br />

of the event and transitioned to MDs of 81 mg/day and<br />

75 mg/day, respectively. He has continual chest pain 2<br />

days after his procedure. Before discharge, platelet reactivity<br />

testing is performed with the VerifyNow P2Y12<br />

assay, resulting in a PRU of 300. Which one of the following<br />

is best to recommend for this patient?<br />

A. Increase clopidogrel dose to 150 mg/day.<br />

B. Reload clopidogrel 600 mg and continue 75 mg/<br />

day.<br />

C. Reload clopidogrel 600 mg and increase the MD<br />

to 150 mg/day.<br />

D. Change to prasugrel 60 mg LD, followed by 10<br />

mg/day.<br />

31. Answer: D<br />

Patients with high post-treatment platelet reactivity have<br />

been shown be at an increased risk of recurrent CV events.<br />

Changing to prasugrel is the best choice because it has been<br />

shown to be superior to standard dosing of clopidogrel in<br />

the management of NSTEMI with invasive management<br />

(Answer D is correct). Although altering the dose of clopidogrel<br />

increases its antiplatelet effects in pharmacodynamic<br />

studies, these strategies have failed to yield improvements<br />

in outcomes (Answer A, Answer B, and Answer C are<br />

incorrect).<br />

1. Cuisset T, Frere C, Quilici J, et al. High post-treatment platelet<br />

reactivity identified low-responders to dual antiplatelet therapy<br />

at increased risk of recurrent cardiovascular events after<br />

stenting for acute coronary syndrome. J Thromb Haemost<br />

2006;4:542-9.<br />

PubMed Link<br />

2. Gurbel PA, Bliden KP, Guyer K, et al. Platelet reactivity in<br />

patients and recurrent events post-stenting: results of the<br />

PREPARE POST-STENTING study. J Am Coll Cardiol<br />

2005;46:1820-6.<br />

PubMed Link<br />

PSAP 2013 • <strong>Cardiology</strong>/<strong>Endocrinology</strong> 13 Answers


3. CURRENT-OASIS 7 Investigators. Dose comparisons of clopidogrel<br />

and aspirin in acute coronary syndromes. N Engl J Med<br />

2010;363:930-42.<br />

PubMed Link<br />

4. Price MJ, Berger PB, Teirstein PS, et al. Standard- vs high-dose<br />

clopidogrel based on platelet function testing after percutaneous<br />

coronary intervention: the GRAVITAS randomized trial.<br />

JAMA 2011;305:1097-105.<br />

PubMed Link<br />

5. Wiviott SD, Braunwald E, McCabe CH, et al. Prasugrel versus<br />

clopidogrel in patients with acute coronary syndromes. N Engl<br />

J Med 2007;357:2001-15.<br />

PubMed Link<br />

32. A 70-year-old woman (weight 70 kg) has a medical history<br />

significant for myocardial infarction (MI) with<br />

subsequent CABG, diabetes, hypertension, and hyperlipidemia.<br />

She presents with left arm pain that has<br />

increased in frequency during the past 48 hours. Her<br />

electrocardiography shows T-wave inversions, and her<br />

laboratory results are hemoglobin 12 g/dL, hematocrit<br />

36%, SCr 0.8 mg/dL, and troponin 8.2 ng/mL. She<br />

does not wish to undergo catheterization at this time.<br />

Which one of the following is the optimal antiplatelet<br />

regimen for this patient?<br />

A. Clopidogrel 600 mg LD, followed by 75 mg/day.<br />

B. Clopidogrel 300 mg LD, followed by 75 mg/day.<br />

C. Prasugrel 60 mg LD, followed by 10 mg/day.<br />

D. Ticagrelor 180 mg LD, followed by 90 mg twice<br />

daily.<br />

32. Answer: D<br />

The patient is being managed noninvasively for NSTEMI;<br />

prasugrel is only appropriate for invasive management<br />

(Answer C is incorrect). Although clopidigrel could be considered<br />

an option for this patient’s medical management,<br />

ticagrelor has been shown to be superior to clopidogrel<br />

for management of NSTEMI, including in patients who<br />

are medically managed. This has led to recommendations<br />

from the American College of Chest Physicians to<br />

recommend ticagrelor over clopidogrel in this patient population<br />

(Answer D is correct; Answer A and Answer B are<br />

incorrect).<br />

1. Wiviott SD, Braunwald E, McCabe CH, et al. Prasugrel versus<br />

clopidogrel in patients with acute coronary syndromes. N Engl<br />

J Med 2007;357:2001-15.<br />

PubMed Link<br />

2. Wallentin L, Becker RC, Budaj A, et al. Ticagrelor versus clopidogrel<br />

in patients with acute coronary syndromes. N Engl J<br />

Med 2009;361:1045-57.<br />

PubMed Link<br />

3. Vandvik PO, Lincoff M, Gore JM, et al. Primary and secondary<br />

prevention of cardiovascular disease: antithrombotic therapy<br />

and prevention of thrombosis, 9th ed: American College of<br />

Chest Physicians evidence-based clinical practice guidelines.<br />

Chest 2012;141:e637S-e668S.<br />

PubMed Link<br />

33. A 72-year-old man (weight 79 kg) has a medical history<br />

significant for MI managed medically, diabetes,<br />

hypertension, and hyperlipidemia. He presents with<br />

angina that has increased in frequency during the past<br />

24 hours. Electrocardiography shows ST depressions,<br />

and laboratory results are hemoglobin 13 g/dL, hematocrit<br />

39%, SCr 1.4 mg/dL, and troponin 7.1 ng/mL.<br />

The patient, who previously underwent genetic testing,<br />

is a noncarrier of the CPY2C19*2 allele. He does not<br />

wish to undergo catheterization at this time. Which<br />

one of the following is the most appropriate antiplatelet<br />

regimen for this patient?<br />

A. Clopidogrel 600 mg LD, followed by 75 mg/day.<br />

B. Prasugrel 60 mg LD, followed by 5 mg/day.<br />

C. Prasugrel 60 mg LD, followed by 10 mg/day.<br />

D. Ticagrelor 180 mg LD, followed by 90 mg twice<br />

daily.<br />

33. Answer: D<br />

This patient is being managed noninvasively for NSTEMI;<br />

prasugrel is only appropriate for invasive management<br />

(Answer B and Answer C are incorrect). Ticagrelor has<br />

been shown to be superior to clopidogrel for management<br />

of NSTEMI, including in patients who are medically managed<br />

(Answer D is correct). However, ticagrelor was also<br />

associated with an increased risk of bleeding events. Despite<br />

genetic profiling, a 300-mg loading dose is preferred in<br />

medical management (Answer A is incorrect).<br />

1. Wiviott SD, Braunwald E, McCabe CH, et al. Prasugrel versus<br />

clopidogrel in patients with acute coronary syndromes. N Engl J<br />

Med 2007;357:2001-15.<br />

PubMed Link<br />

2. Wallentin L, Becker RC, Budaj A, et al. Ticagrelor versus clopidogrel<br />

in patients with acute coronary syndromes. N Engl J Med<br />

2009;361:1045-57.<br />

PubMed Link<br />

3. Yusef S, Zhao F, Mehta SR, et al. The Clopidogrel in Unstable<br />

Angina to Prevent Recurrent Event Trial Investigators. Effects<br />

of clopidogrel in addition to aspirin in patients with acute coronary<br />

syndromes without ST-segment elevation. N Engl J Med<br />

2001;345:494-502.<br />

PubMed Link<br />

4. Mega JL, Close SL, Wiviott SD, et al. Cytochrome P-450<br />

polymorphisms and response to clopidogrel. N Engl J Med<br />

2009;360:354-62.<br />

PubMed Link<br />

5. Holmes MV, Perel P, Shah T, et al. CYP2C19 genotype, clopidogrel<br />

metabolism, platelet function, and cardiovascular events: a<br />

systematic review and meta-analysis. JAMA 2011;306:2704-14.<br />

PubMed Link<br />

Answers<br />

14<br />

PSAP 2013 • <strong>Cardiology</strong>/<strong>Endocrinology</strong>


34. A 58-year-old man (weight 70 kg) presents with anterior<br />

STEMI treated with a bare metal stent (BMS) to<br />

his left anterior descending coronary artery. He is initiated<br />

on aspirin 81 mg and clopidogrel 600 mg LD,<br />

followed by 75 mg/day. The following morning, echocardiography<br />

shows a reduced left ventricular (LV)<br />

ejection fraction of 30% and the presence of an LV<br />

thrombus. In addition to continuing aspirin, which one<br />

of the following is best to recommend for this patient?<br />

A. Add warfarin (INR 2–3) and clopidogrel for 1<br />

month only.<br />

B. Add warfarin (INR 2–2.5) and clopidogrel for 3<br />

months only.<br />

C. Add warfarin (INR 2–3) and clopidogrel for 3<br />

months only.<br />

D. Add warfarin (INR 2–3) for 3 months and discontinue<br />

clopidogrel.<br />

34. Answer: A<br />

This patient received a BMS after STEMI and now has<br />

an LV thrombus requiring anticoagulation in addition to<br />

antiplatelet therapy. Answer B is incorrect because this recommendation<br />

comes from the ACC/AHA guidelines for<br />

NSTEMI management in patients with an indication for<br />

anticoagulation. Answer C and Answer D are incorrect<br />

because these are American College of Chest Physicians<br />

guideline recommendations for STEMI patients who have<br />

received DES or no stents, respectively. Answer A is correct<br />

because it is consistent with American College of Chest<br />

Physicians guidelines.<br />

1. Anderson JL, Adams CD, Antman EM, et al. 2011 ACCF/<br />

AHA Focused Update Incorporated Into the ACC/AHA 2007<br />

Guidelines for the Management of Patients with Unstable<br />

Angina/Non-ST-Elevation Myocardial Infarction: a report of<br />

the American College of <strong>Cardiology</strong> Foundation/American<br />

Heart Association Task Force on Practice Guidelines.<br />

Circulation 2011;123:e426-e-579.<br />

PubMed Link<br />

2. Vandvik PO, Lincoff M, Gore JM, et al. Primary and secondary<br />

prevention of cardiovascular disease: antithrombotic therapy<br />

and prevention of thrombosis, 9th ed: American College of<br />

Chest Physicians evidence-based clinical practice guidelines.<br />

Chest 2012;141:e637S-e668S.<br />

PubMed Link<br />

35. Which one of the following best describes a pharmacodynamic<br />

benefit of prasugrel over clopidogrel?<br />

A. No requirement for activation to exert its effects.<br />

B. Improved selectivity for the P2Y12 receptor.<br />

C. Decreased variability in antiplatelet effect.<br />

D. Lack of CYP3A metabolism.<br />

35. Answer: C<br />

Like clopidogrel, prasugrel is a prodrug that requires activation<br />

through a CYP-dependent process (Answer A is<br />

incorrect). Both drugs are metabolized by CYP3A and<br />

do not have differences in binding to the P2Y12 receptor<br />

(Answers B and Answer D are incorrect). Answer C is correct<br />

because prasugrel has been consistently shown to have<br />

more potent and consistent antiplatelet effects.<br />

1. Jakubowski JA, Matsushima N, Asai F, et al. A multiple dose<br />

study of prasugrel (CS-747), a novel thienopyridine P2Y12<br />

inhibitor, compared with clopidogrel in healthy humans. Br J<br />

Clin Pharmacol 2007;63:421-30.<br />

PubMed Link<br />

2. Holmes MV, Perel P, Shah T, et al. CYP2C19 genotype,<br />

clopidogrel metabolism, platelet function, and cardiovascular<br />

events: a systematic review and meta-analysis. JAMA<br />

2011;306:2704-14.<br />

PubMed Link<br />

36. A 65-year-old man (weight 79 kg) presents to the<br />

emergency department with chest pain, ST-segment<br />

depressions on electrocardiography, and elevated troponins.<br />

The patient’s medical history includes asthma,<br />

hypertension, and end-stage kidney disease after<br />

kidney transplantation. His drugs include amlodipine,<br />

aspirin, ketoconazole, mycophenolate mofetil,<br />

omeprazole, pravastatin, and tacrolimus. Which one of<br />

the following therapy changes would be most appropriate<br />

if clopidogrel were added to treat this patient’s<br />

ACS event?<br />

A. Change amlodipine to diltiazem.<br />

B. Change pravastatin to atorvastatin.<br />

C. Change ketoconazole to voriconazole.<br />

D. Change omeprazole to pantoprazole.<br />

36. Answer: D<br />

Although amlodipine and ketoconazole coadministration<br />

have been linked with reductions in antiplatelet effects of<br />

clopidogrel, no data link them to worse clinical outcomes<br />

(Answer A and Answer C are incorrect). Additionally,<br />

changing amlodipine to diltiazem could lead to an increase<br />

in tacrolimus serum concentrations because diltiazem may<br />

inhibit the metabolism of tacrolimus. Answer B is incorrect<br />

in that atorvastatin has been reported to increase cardiovascular<br />

events when combined with clopidogrel; however,<br />

these reports are inconsistent. Pantoprazole is one of the<br />

PPIs that has not been linked with adverse CV events in any<br />

of the reported analyses (Answer D is correct).<br />

1. Farid NA, Payne CD, Small DC, et al. Cytochrome P450 3A<br />

inhibition by ketoconazole affects prasugrel and clopidogrel<br />

pharmacokinetics and pharmacodynamics differently. Clin<br />

Pharmacol Ther 2007;81:735-41.<br />

PubMed Link<br />

PSAP 2013 • <strong>Cardiology</strong>/<strong>Endocrinology</strong> 15 Answers


2. Gremmel T, Steiner S, Seidinger D, et al. Calcium-channel<br />

blockers decrease clopidogrel-mediated platelet inhibition.<br />

Heart 2010;96:186-9.<br />

PubMed Link<br />

3. Bates ER, Lau WC, Angiolillo DJ. Clopidogrel-drug interactions.<br />

J Am Coll Cardiol 2011;57:1251-63.<br />

PubMed Link<br />

37. You are asked to provide a recommendation for a<br />

formulary decision on clopidogrel, prasugrel, and<br />

ticagrelor for use in your institution in patients admitted<br />

with ACS. Which one of the following strategies is<br />

most cost-effective given the currently available data?<br />

A. Provide universal use of prasugrel for all patients.<br />

B. Genotype patients and use prasugrel for those with<br />

loss-of-function (LOF) alleles for clopidogrel.<br />

C. Provide universal use of ticagrelor for all patients.<br />

D. Genotype patients and use ticagrelor for those<br />

with LOF alleles for clopidogrel.<br />

37. Answer: C<br />

Answer A and Answer B are incorrect because while both<br />

universal and genotype-directed prasugrel have been found<br />

to be cost-effective compared with clopidogrel, they have<br />

not been shown to be better than ticagrelor. Universal<br />

ticagrelor was shown to be better than genotype-based<br />

ticagrelor (Answer C is correct; Answer D is incorrect).<br />

1. Mahoney EM, Wang K, Arnold SV, et al. Cost-effectiveness of<br />

prasugrel versus clopidogrel in patients with acute coronary<br />

syndromes and planned percutaneous coronary intervention:<br />

results from the trial to assess improvement in therapeutic<br />

outcomes by optimizing platelet inhibition with Prasugrel-<br />

Thrombolysis in Myocardial Infarction TRITON-TIMI 38.<br />

Circulation 2010;121:71-9.<br />

PubMed Link<br />

2. Lee S, Dhamija A, Parsons K, et al. Cost-effectiveness of universal<br />

or genotype driven use of available dual antiplatelet<br />

strategies in patients with acute coronary syndrome [abstract].<br />

J Am Coll Cardiol 2012;59:E483.<br />

Internet Link<br />

3. Crespin DJ, Federspiel JJ, Biddle AK, et al. Ticagrelor versus<br />

genotype-driven antiplatelet therapy for secondary prevention<br />

after acute coronary syndrome: a cost-effectiveness analysis.<br />

Value in Health 2011;14:483-91.<br />

PubMed Link<br />

4. Reese ES, Mullins D, Beitelshees AL, et al. Cost-effectiveness<br />

of cytochrome P450 2C19 genotype screening for selection<br />

of antiplatelet therapy with clopidogrel or prasugrel.<br />

Pharmacotherapy 2012;32:323-32.<br />

PubMed Link<br />

38. A 55-year-old man (weight 75 kg) presents to the<br />

emergency department with chest pain, ST-segment<br />

depressions on electrocardiography, and an increased<br />

troponin level of 4.5 ng/mL. Laboratory results are<br />

hemoglobin 12 g/dL, hematocrit 36%, and SCr 1.0<br />

mg/dL. His medical history is significant for AF<br />

(annual stroke risk < 3% per year), upper gastrointestinal<br />

bleed, and hypertension. The patient’s oral drugs<br />

are chlorthalidone 25 mg daily, metoprolol tartrate 50<br />

mg twice daily, and warfarin 5 mg daily. He receives<br />

a DES, and he is treated with aspirin 81 mg/day and<br />

clopidogrel 75 mg/day. Which one of the following<br />

changes to the patient’s home drugs would be best<br />

before his discharge?<br />

A. Add omeprazole.<br />

B. Add pantoprazole.<br />

C. Add ranitidine.<br />

D. Discontinue warfarin.<br />

38. Answer: D<br />

The patient has been treated with DES placement after<br />

NSTEMI and has atrial fibrillation on warfarin therapy<br />

before admission. In light of the patient’s low risk of stroke<br />

from atrial fibrillation (hypertension being his only risk<br />

factor), he does not require anticoagulation, particularly<br />

in light of his gastrointestinal bleeding history. Although<br />

Answer A, Answer B, and Answer C represent options for<br />

potenitally reducing the risk of gastrointestinal bleeding in<br />

patients requiring triple antithrombotic therapy, they are all<br />

incorrect because this patient has a low risk of stroke and a<br />

high risk for increased bleeding, making Answer D correct.<br />

1. Vandvik PO, Lincoff M, Gore JM, et al. Primary and secondary<br />

prevention of cardiovascular disease: antithrombotic therapy<br />

and prevention of thrombosis, 9th ed: American College of<br />

Chest Physicians evidence-based clinical practice guidelines.<br />

Chest 2012;141:e637S-e68S.<br />

PubMed Link<br />

2. Wann LS, Curtis AB, January CT, et al. 2011 ACCF/AHA/<br />

HRS focused update on the management of patients with<br />

atrial fibrillation (updating the 2006 guideline): a report of the<br />

American College of <strong>Cardiology</strong> Foundation/American Heart<br />

Association Task Force on Practice Guidelines. Circulation<br />

2011;123:104-23.<br />

PubMed Link<br />

39. Which one of the following patients treated for ACS<br />

would most likely experience harm if treated with prasugrel<br />

over clopidogrel therapy?<br />

A. A 73-year-old patient (weight 70 kg) with a history<br />

of TIA treated for NSTEMI with DES<br />

implantation.<br />

B. A 75-year-old patient with diabetes (weight 80 kg)<br />

treated for STEMI with DES implantation.<br />

Answers<br />

16<br />

PSAP 2013 • <strong>Cardiology</strong>/<strong>Endocrinology</strong>


C. A 70-year-old patient (weight 90 kg) treated for<br />

NSTEMI with medical management.<br />

D. A 71-year-old patient (weight 58 kg) treated for<br />

NSTEMI with BMS implantation.<br />

39. Answer: A<br />

Patients aged 75 years or older or weighing 60 kg or under<br />

were found to have no net benefit with prasugrel therapy,<br />

but also no net harm (Answer B and Answer D are incorrect).<br />

The medically managed subgroup was found to<br />

have an insignificant difference in outcomes with prasugrel<br />

but again no net harm (Answer C is incorrect). The<br />

only subgroup found to have experienced net harm in the<br />

TRITON-TIMI 38 trial was patients with history of stroke<br />

or TIA (Answer A is correct).<br />

1. Wiviott SD, Braunwald E, McCabe CH, et al. Prasugrel versus<br />

clopidogrel in patients with acute coronary syndromes. N Engl<br />

J Med 2007;357:2001-15.<br />

PubMed Link<br />

2. Pride YB, Wiviott SD, Buros JL, et al. Effect of prasugrel versus<br />

clopidogrel on outcomes among patients with acute coronary<br />

syndrome undergoing percutaneous coronary intervention<br />

without stent implantation: a TRial to assess Improvement in<br />

Therapeutic Outcomes by optimizing platelet inhibitioN with<br />

prasugrel (TRITON)-Thrombolysis in Myocardial Infarction<br />

(TIMI) 38 substudy. Am Heart J 2009;158:e21-6.<br />

PubMed Link<br />

40. Which one of the following patients would have the<br />

greatest risk of death or reinfarction after an ACS<br />

event?<br />

A. A 65-year-old man with ST depressions on electrocardiography,<br />

troponin 12.2 ng/mL, and<br />

previously diagnosed 40% stenosis in the right coronary<br />

artery.<br />

B. A 70-year-old woman with ST depressions on electrocardiography,<br />

negative troponin, and reported<br />

aspirin use until the day of admission.<br />

C. A 60-year-old man with ST depressions on electrocardiography,<br />

troponin 17.1 ng/mL, and reported<br />

nitroglycerin use within 7 days.<br />

D. A 62-year-old woman with ST depressions on<br />

electrocardiography, troponin 11.5 ng/mL, previously<br />

diagnosed 60% stenosis in the left anterior<br />

descending artery, and reported aspirin use until<br />

the day of admission.<br />

40. Answer: D<br />

Based upon the TIMI risk score, the following have been<br />

identified as risk factors for death, MI, or urgent revascularization:<br />

age 65 years or greater; three or more CAD risk<br />

factors; prior coronary stenosis of 50% or greater; aspirin<br />

use within 7 days; elevated biomarkers; and ST deviation.<br />

That makes the patient in Answer D the correct response.<br />

The patients described in Answer A, Answer B, and Answer<br />

C have lower risk.<br />

1. Antman EM, Cohen M, Bernink PJ, et al. The TIMI risk<br />

score for unstable angina/non-ST elevation MI: a method<br />

for prognostication and therapeutic decision making. JAMA<br />

2000;284:835-42.<br />

PubMed Link<br />

2. Jneid H, Anderson JL, Wright RS, et al. 2012 ACCF/AHA<br />

focused update of the guideline for the management of patients<br />

with unstable angina/Non–ST-elevation myocardial infarction<br />

(updating the 2007 guideline and replacing the 2011 focused<br />

update): a report of the American College of <strong>Cardiology</strong><br />

Foundation/American Heart Association Task Force on practice<br />

guidelines. Circulation 2012;126:875-910.<br />

PubMed Link<br />

3. Kushner SG, Hand M, Smith SC Jr, et al. 2009 Focused Updates:<br />

ACC/AHA Guidelines for the Management of Patients<br />

With ST-Elevation Myocardial Infarction (updating the 2004<br />

Guideline and 2007 Focused Update) and ACC/AHA/SCAI<br />

Guidelines on Percutaneous Coronary Intervention (updating<br />

the 2005 Guideline and 2007 Focused Update): a report of the<br />

American College of <strong>Cardiology</strong> Foundation/American Heart<br />

Association Task Force on Practice Guidelines. Circulation<br />

2009;120:2271-306.<br />

PubMed Link<br />

Newer Anticoagulation Strategies<br />

in Atrial Fibrillation<br />

Questions 41–43 pertain to the following case<br />

M.M. is a 67-year-old man with a history of hypertension,<br />

diabetes, and mitral stenosis secondary to rheumatic<br />

heart disease. He presents to the cardiology clinic for follow-up.<br />

His heart rate is 82 beats/minute, but the rhythm<br />

is determined to be irregularly irregular during physical<br />

examination. Electrocardiography reveals atrial fibrillation<br />

(AF). On previous echocardiography, M.M. was found<br />

to have a dilated left atrium secondary to mitral stenosis;<br />

this is currently being medically managed. Results of his<br />

complete blood cell count (CBC), activated partial thromboplastin<br />

time (aPTT), PT/INR, and basic metabolic panel<br />

(BMP) are within normal limits, and his fasting blood glucose<br />

is 123 mg/dL. He currently takes aspirin 325 mg/day,<br />

simvastatin 40 mg at bedtime, lisinopril 20 mg/day, and<br />

metformin 1000 mg twice daily.<br />

41. Given his medical history, which one of the following is<br />

M.M.’s strongest single risk factor for stroke as a result<br />

of AF?<br />

A. Age.<br />

B. Mitral stenosis.<br />

C. Diabetes.<br />

D. Hypertension.<br />

PSAP 2013 • <strong>Cardiology</strong>/<strong>Endocrinology</strong> 17 Answers


41. Answer: B<br />

Mitral stenosis is considered a major risk factor for stroke<br />

according to ACC/AHA/HRS and <strong>ACCP</strong> atrial fibrillation<br />

(AF) guidelines (Answer B is correct). Answer A, Answer<br />

C, and Answer D are considered moderate risk factors and<br />

are therefore incorrect as single risk factors.<br />

1. Yamamoto K, Ikeda U, Seino Y, et al. Coagulation activity is<br />

increased in the left atrium of patients with mitral stenosis. J<br />

Am Coll Cardiol 1995;25:107-12.<br />

PubMed Link<br />

2. You JJ, Dinger DE, Howard PA, et al. Antithrombotic therapy<br />

for atrial fibrillation: antithrombotic therapy and<br />

prevention of therombosis, 9th edition: American College of<br />

Chest Physicians Evidence-Based Clinical Practice Guidelines.<br />

Chest 2012; 141(suppl 2); e531S-e575S.<br />

PubMed Link<br />

42. Which one of the following best describes M.M.’s risk<br />

of ischemic stroke secondary to AF?<br />

A. Extremely low.<br />

B. Low.<br />

C. Intermediate.<br />

D. High.<br />

42. Answer: D<br />

Based on either his CHADS 2<br />

score of 2 or greater or the<br />

presence of mitral stenosis alone, this patient would be classified<br />

as having a high risk of stroke (Answer D is correct).<br />

Extremely low is not a recognized categorization of stroke<br />

risk in AF (Answer A is incorrect). Answer B is incorrect<br />

because a CHADS 2<br />

score of 0 would equate to low risk.<br />

Answer C is therefore also incorrect because a CHADS 2<br />

score of 1 would equate to intermediate risk.<br />

1. You JJ, Dinger DE, Howard PA, et al. Antithrombotic therapy<br />

for atrial fibrillation: antithrombotic therapy and<br />

prevention of therombosis, 9th edition: American College of<br />

Chest Physicians Evidence-Based Clinical Practice Guidelines.<br />

Chest 2012; 141(suppl 2); e531S-e575S.<br />

PubMed Link<br />

2. Yamamoto K, Ikeda U, Seino Y, et al. Coagulation activity is<br />

increased in the left atrium of patients with mitral stenosis. J<br />

Am Coll Cardiol 1995;25:107-12.<br />

PubMed Link<br />

43. Which one of the following antithrombotic regimens<br />

would be best to recommend for M.M. to prevent ischemic<br />

stroke caused by AF?<br />

A. Aspirin plus clopidogrel.<br />

B. Dabigatran.<br />

C. Rivaroxaban.<br />

D. Warfarin.<br />

43. Answer: D<br />

Answer D is correct because the patient has mitral stenosis,<br />

and it is strongly indicated that the origin of the patient’s AF<br />

is valvular. Warfarin is the only drug recommended in this<br />

scenario. Answer A is incorrect because the patient’s stroke<br />

risk, in the absence of contraindications, warrants the use of<br />

anticoagulant therapies because of the superior risk reduction<br />

versus antiplatelet therapies. Answer B and Answer<br />

C are incorrect because patients with valvular AF were<br />

excluded from clinical analysis with these therapies, and<br />

these agents should not be considered first-line options.<br />

1. You JJ, Dinger DE, Howard PA, et al. Antithrombotic therapy<br />

for atrial fibrillation: antithrombotic therapy and<br />

prevention of therombosis, 9th edition: American College of<br />

Chest Physicians Evidence-Based Clinical Practice Guidelines.<br />

Chest 2012; 141(suppl 2); e531S-e575S.<br />

PubMed Link<br />

2. Yamamoto K, Ikeda U, Seino Y, et al. Coagulation activity is<br />

increased in the left atrium of patients with mitral stenosis. J<br />

Am Coll Cardiol 1995;25:107-12.<br />

PubMed Link<br />

Questions 44–47 pertain to the following case.<br />

L.D., a 68-year-old woman with a history of permanent AF<br />

after a failed ablation, currently receives rate-controlling<br />

therapy. She has early-onset mild Alzheimer dementia as<br />

well as a history of chronic obstructive pulmonary disease,<br />

tobacco abuse, and medication nonadherence. She lives<br />

alone and has regular home health assistance in preparing<br />

her pillbox for the week. Her current drugs are warfarin 5<br />

mg/day alternating with 7.5 mg (current INR 2.6), donepezil<br />

5 mg/day, aspirin 81 mg/day, tiotropium 18 mcg/day,<br />

and fluticasone/salmeterol 250/50 twice daily. Because of<br />

her dementia, she depends on her pillbox as a memory aid.<br />

She has difficulty in getting to her anticoagulation clinic<br />

appointments and in maintaining a consistent diet, leading<br />

to the necessity of frequent monitoring of her INR.<br />

However, her previous 6 months of anticoagulation visits<br />

have demonstrated excellent INR control. Her CBC and<br />

BMP are within normal limits.<br />

44. Which one of the following best describes L.D.’s stroke<br />

risk, a consequence of AF, using the CHADS 2<br />

and the<br />

CHA 2<br />

DS 2<br />

VASc scores, respectively?<br />

A. Low, intermediate.<br />

B. Low, high.<br />

C. High, high.<br />

D. Intermediate, high.<br />

44. Answer: B<br />

The patient has none of the risk factors that are components<br />

of the CHADS 2<br />

score. Therefore, her score would be zero,<br />

and her risk assessment would be considered low (Answer<br />

C and Answer D are incorrect). This patient has two risk<br />

Answers<br />

18<br />

PSAP 2013 • <strong>Cardiology</strong>/<strong>Endocrinology</strong>


factors according to the CHA 2<br />

DS 2<br />

VASc system: female sex<br />

and age older than 65. A score of 2 or greater according to<br />

the HA 2<br />

DS 2<br />

VASc system is considered a high risk of stroke<br />

(Answer A is incorrect, Answer B is correct).<br />

1. Odum LE, Cochran KA, Aistrope DS, et al. The CHADS(2)<br />

versus the new CHA(2) DS(2) -VASc scoring systems for<br />

guiding antithrombotic treatment of patients with atrial fibrillation:<br />

review of the literature and recommendations for use.<br />

Pharmacotherapy 2012;32:285-96.<br />

PubMed Link<br />

2. You JJ, Dinger DE, Howard PA, et al. Antithrombotic therapy<br />

for atrial fibrillation: antithrombotic therapy and<br />

prevention of therombosis, 9th edition: American College of<br />

Chest Physicians Evidence-Based Clinical Practice Guidelines.<br />

Chest 2012; 141(suppl 2); e531S-e575S.<br />

PubMed Link.<br />

45. Which one of the following is the best option to<br />

decrease L.D.’s risk of bleeding from antithrombotic<br />

therapy?<br />

A. Discontinue warfarin; initiate rivaroxaban 20 mg/<br />

day.<br />

B. Continue warfarin; target an INR of 1.6–2.5.<br />

C. Continue warfarin at current dose; discontinue<br />

aspirin.<br />

D. Discontinue warfarin; add clopidogrel to aspirin<br />

therapy.<br />

45. Answer: C<br />

This patient does not appear to have an indication for concomitant<br />

use of aspirin and anticoagulant therapy, and the<br />

use of aspirin plus warfarin represents a pharmacodynamic<br />

drug-drug interaction, which increases the risk of bleeding<br />

events (Answer C is correct). Answer A may be an option if<br />

the patient was having frequent supratherapeutic INR values,<br />

but is less important at this point than Answer C because<br />

her INR today is currently within the therapeutic range at<br />

2.6 and she has demonstrated excellent INR control over the<br />

previous 6 months. Additionally, the patient’s history of nonadherence<br />

and known mild dementia would not be optimal<br />

circumstances for pursuing a narrower INR range (Answer A<br />

is incorrect; Answer C is correct). Answer B is a recommendation<br />

from the ACC/AHA/HRS AF guidelines for patients<br />

at high risk of bleeding and age greater than 75. However,<br />

this patient does not have specific high-risk features associated<br />

with bleeding and is younger than 75 years. Answer D<br />

is incorrect given that the combination of aspirin and clopidogrel<br />

is inferior to warfarin in the prevention of stroke and<br />

would not change the risk of bleeding.<br />

1. Hansen ML, Sørensen R, Clausen MT, et al. Risk of bleeding<br />

with single, dual, or triple therapy with warfarin, aspirin, and<br />

clopidogrel in patients with atrial fibrillation. Arch Intern Med<br />

2010;170:1433-41.<br />

PubMed Link<br />

2. Wann LS, Curtis AB, January CT, et al. 2011 ACCF/AHA/<br />

HRS focused update on the management of patients with atrial<br />

fibrillation (updating the 2006 guideline). J Am Coll Cardiol<br />

2011;57:223-42.<br />

PubMed Link<br />

46. Considering her CHA 2<br />

DS 2<br />

VASc score, which one of<br />

the following antithrombotic options would best help<br />

L.D. decrease the number of clinic visits, given the difficulties<br />

she has with transportation?<br />

A. Discontinue warfarin; add clopidogrel to aspirin<br />

therapy.<br />

B. Continue warfarin, but change the INR goal to<br />

1.6–2.5.<br />

C. Discontinue warfarin; add dabigatran 150 mg<br />

twice daily.<br />

D. Discontinue warfarin; add rivaroxaban 20 mg<br />

once daily.<br />

46. Answer: D<br />

The combination of aspirin and clopidogrel is inferior to<br />

warfarin in stroke prevention in AF and should only be considered<br />

in patients who are not candidates for anticoagulant<br />

therapy (Answer A is incorrect). Answer B could be considered<br />

to lower this patient’s risk of bleeding, but the narrower<br />

INR range will likely increase her need for clinic visits and<br />

increase monitoring (Answer B is incorrect). Answer C is<br />

also reasonable, but the patient appears to rely heavily on<br />

a memory aid with a pillbox and has a history of nonadherence.<br />

Dabigatran cannot be removed from the product<br />

container and placed into such a device (Answer A is incorrect,<br />

Answer D is correct).<br />

1. Wann LS, Curtis AB, January CT, et al. 2011 ACCF/AHA/<br />

HRS focused update on the management of patients with atrial<br />

fibrillation (updating the 2006 guideline). J Am Coll Cardiol<br />

2011;57:223-42.<br />

PubMed Link<br />

2. Spinler SA, Willey VJ. A patient’s guide to taking dabigatran<br />

etexilate. Circulation 2011;124:e209-e211.<br />

PubMed Link<br />

47. L.D. shows interest in testing her INR at home after<br />

reading about it in “Anticoagulation Weekly.” Her<br />

Medicare plan will provide reimbursement for the<br />

cost of the device and testing strips. Which one of the<br />

following represents the most significant barrier to a<br />

self-testing strategy for L.D.’s warfarin?<br />

A. A likely increase in the number of her clinic visits<br />

over time.<br />

B. Increasing drug acquisition costs for her warfarin.<br />

PSAP 2013 • <strong>Cardiology</strong>/<strong>Endocrinology</strong> 19 Answers


C. Reimbursing the anticoagulation provider for<br />

telephonic management.<br />

D. Fulfilling competency requirements.<br />

47. Answer: D<br />

This patient’s history of nonadherence, reliance on home<br />

health to assist in drug administration, and presence of<br />

dementia make her a poor candidate for fulfilling competency<br />

requirements to use the device (Answer D is<br />

correct). Answer A is incorrect because it is likely clinic<br />

visits will decrease over time with self-testing, although<br />

they may increase initially. Reimbursement for telephonic<br />

management is also likely not a significant barrier given<br />

that many anticoagulation practitioners recommend and<br />

follow patients who are self-testing despite the lack of program<br />

reimbursement (Answer C is incorrect). Answer B<br />

is also incorrect because of the relatively low drug acquisition<br />

costs associated with warfarin; it is also unlikely that<br />

any change in testing strategy would significantly alter drug<br />

expenditure with warfarin.<br />

1. Bussey HI, Bussey M. <strong>Cardiology</strong> patient page. Warfarin<br />

management: international normalized ratio self-testing and<br />

warfarin self-dosing. Circulation 2012;126:e52-4.<br />

PubMed Link<br />

2. Bloomfield HE, Krause A, Greer, et al. Meta-analysis: Effect<br />

of patient self-testing and self-management of long-term anticoagulation<br />

on major clinical outcomes. Ann Intern Med<br />

2011;154:472-82.<br />

PubMed Link<br />

48. Compared with usual management, which one of the<br />

following best describes the utility of patient self-testing for<br />

warfarin?<br />

A. It achieves superior time in therapeutic range.<br />

B. It achieves superior clinical outcomes.<br />

C. It improves quality of life.<br />

D. Devices are expensive and not reimbursed by<br />

insurance.<br />

48. Answer: C<br />

In the THINRS study, the largest randomized trial of selftesting<br />

versus usual management, self-testing demonstrated<br />

statistically significant time in therapeutic range (TTR) compared<br />

with usual care. However, both groups had TTR in the<br />

60% range, which likely indicates that the difference is not<br />

clinically significant (Answer A is incorrect). Furthermore,<br />

THINRS did not demonstrate any significant difference in<br />

clinical outcomes (Answer B is incorrect). However, quality<br />

of life was significantly improved in patients receiving<br />

self-testing versus usual care (Answer C is correct). Answer<br />

D is incorrect because self-testing devices are reimbursed<br />

by many insurance plans.<br />

1. Gadisseur AP, Breukink-Engbers WG, van der Meer FJ, et<br />

al. Comparison of the quality of oral anticoagulant therapy<br />

through patient self-management and management by specialized<br />

anticoagulation clinics in the Netherlands: a randomized<br />

clinical trial. Arch Intern Med 2003;163:2639-46.<br />

PubMed Link<br />

2. Gardiner C, Williams K, Longair I, et al. A randomised<br />

control trial of patient self-management of oral anticoagulation<br />

compared with patient self-testing. Br J Haematol<br />

2006;132:598-603.<br />

PubMed Link<br />

Questions 49–51 pertain to the following case.<br />

T.W. is a 78-year-old man with a history of paroxysmal AF,<br />

currently rhythm controlled on dronedarone. Despite several<br />

trials of antiarrhythmic drug therapy he continues<br />

to experience palpitations; therefore he is scheduled to<br />

undergo left atrial ablation later in the week. His medical<br />

history includes hypertension, hyperlipidemia, depression,<br />

and osteoarthritis. Currently, T.W. is receiving no anticoagulant<br />

therapy after experiencing a hemopneumothorax on<br />

warfarin after a car crash. His electrophysiologist will perform<br />

transesophageal echocardiography before ablation.<br />

T.W.’s drug regimen is aspirin 81 mg/day, atorvastatin 20<br />

mg/day, dronedarone 400 mg twice daily, and citalopram<br />

20 mg/day.<br />

49. Which one of the following would best prevent preprocedure<br />

and inter-procedure thrombus formation<br />

and minimize the bleeding risk during left atrial ablation<br />

for T.W.?<br />

A. Start warfarin as an adjunct to procedural<br />

therapies.<br />

B. Start dabigatran and hold the morning dose<br />

before the procedure.<br />

C. Continue aspirin and administer inter-procedural<br />

heparin.<br />

D. Start dabigatran and give it without interruption<br />

through the procedure.<br />

49. Answer: C<br />

Answer A would be reasonable to continue if the patient<br />

was already receiving warfarin, but it is not recommended<br />

that warfarin be initiated in a patient naïve to therapy before<br />

ablation (Answer A is incorrect). Answer B and Answer D<br />

are incorrect because the use of dabigatran before ablation<br />

procedures is associated with worsened clinical outcomes,<br />

even when the morning dose is held. Answer C is correct<br />

because intra-procedural anticoagulant therapy is necessary<br />

for preventing inter-procedure thrombus formation,<br />

even if preprocedural echocardiography is performed. This<br />

patient’s current antithrombotic prophylaxis should be<br />

continued pre-procedure.<br />

Answers<br />

20<br />

PSAP 2013 • <strong>Cardiology</strong>/<strong>Endocrinology</strong>


1. Wann LS, Curtis AB, January CT, et al. 2011 ACCF/AHA/<br />

HRS focused update on the management of patients with atrial<br />

fibrillation (updating the 2006 guideline). J Am Coll Cardiol<br />

2011;57:223-42.<br />

PubMed Link<br />

2. You JJ, Dinger DE, Howard PA, et al. Antithrombotic therapy<br />

for atrial fibrillation: antithrombotic therapy and<br />

prevention of therombosis, 9th edition: American College of<br />

Chest Physicians Evidence-Based Clinical Practice Guidelines.<br />

Chest 2012; 141(suppl 2); e531S-e575S.<br />

PubMed Link<br />

50. The procedure is successful. Which one of the following<br />

is the optimal length of anticoagulant therapy after<br />

left atrial ablation for T.W.?<br />

A. 1 month.<br />

B. 2 months.<br />

C. 1 year.<br />

D. Indefinitely.<br />

50. Answer: D<br />

This patient has a CHADS 2<br />

score of 2, indicating a high<br />

risk of stroke. A minimum duration of 2 months of anticoagulation<br />

after left atrial ablation is indicated, with the risk<br />

thereafter to be determined by the CHADS 2<br />

score. Given<br />

the patient’s high risk of stroke, Answer D is correct. The<br />

minimum duration of anticoagulation recommended postablation<br />

(2 months) could be considered given that he<br />

was not receiving anticoagulation before the procedure.<br />

However, the risk of bleeding in this patient appears to be<br />

attributable to a traumatic event in the setting of anticoagulation<br />

and does not represent a contraindication to current<br />

treatment (Answer B is incorrect). Durations of 1 month<br />

and 1 year are not timeframes that are addressed in the management<br />

of antithrombotic therapy with ablation (Answer<br />

A and Answer C are incorrect).<br />

1. Wann LS, Curtis AB, January CT, et al. 2011 ACCF/AHA/<br />

HRS focused update on the management of patients with atrial<br />

fibrillation (updating the 2006 guideline). J Am Coll Cardiol<br />

2011;57:223-42.<br />

PubMed Link<br />

2. You JJ, Dinger DE, Howard PA, et al. Antithrombotic therapy<br />

for atrial fibrillation: antithrombotic therapy and<br />

prevention of therombosis, 9th edition: American College of<br />

Chest Physicians Evidence-Based Clinical Practice Guidelines.<br />

Chest 2012; 141(suppl 2); e531S-e575S.<br />

PubMed Link<br />

51. Two weeks after ablation, T.W. reports to the clinic<br />

feeling well. He has continued taking dronedarone,<br />

and after his ablation, he was placed on dabigatran 150<br />

mg twice daily. He reports significant reflux that his<br />

primary physician told him is likely caused by dabigatran.<br />

The physician recommended omeprazole 40 mg/<br />

day, which has not helped significantly. Which one of<br />

the following is best to recommend for T.W.?<br />

A. Continue dabigatran; increase omeprazole to 40<br />

mg twice daily.<br />

B. Discontinue dabigatran; start warfarin with target<br />

INR 1.6–2.5.<br />

C. Discontinue dabigatran; start rivaroxaban 20 mg/<br />

day.<br />

D. Discontinue dabigatran; start aspirin 81 mg/day<br />

plus clopidogrel 75 mg/day.<br />

51. Answer: C<br />

Answer C is correct because this likely represents an<br />

adverse reaction from the dabigatran and probably should<br />

not be continued. Given that the patient is still within the<br />

2-month window of risk from the ablation procedure, simply<br />

starting warfarin without a parenteral bridge will likely<br />

result in a period of subtherapeutic anticoagulation and<br />

is not recommended; the INR range is also unnecessarily<br />

restrictive (Answer B is incorrect). Answer A likely will<br />

not be very effective; additionally, the potential of a drug<br />

interaction with intensive acid-suppressive therapy and the<br />

combination of a p-glycoprotein inhibitor with dronedarone<br />

is concerning (Answer A is incorrect). Answer D is<br />

incorrect because aspirin plus clopidogrel are inferior to<br />

anticoagulant therapy in stroke prevention in AF and are<br />

not recommended after ablation.<br />

2. You JJ, Dinger DE, Howard PA, et al. Antithrombotic therapy<br />

for atrial fibrillation: antithrombotic therapy and<br />

prevention of therombosis, 9th edition: American College of<br />

Chest Physicians Evidence-Based Clinical Practice Guidelines.<br />

Chest 2012; 141(suppl 2); e531S-e575S.<br />

PubMed Link<br />

2. Hankey GJ, Eikelboom JW. Dabigatran etexilate: a new oral<br />

thrombin inhibitor. Circulation 2011: 123:1436-50.<br />

PubMed Link<br />

Questions 52–54 pertain to the following case.<br />

Y.G. is a 68-year-old man with hypertension, hemodialysis-dependent<br />

end-stage kidney disease, and a history of<br />

hepatitis C with moderate-severe liver dysfunction. He<br />

was given a diagnosis of AF after a recent admission for a<br />

gastrointestinal bleed, but he reports being asymptomatic.<br />

His laboratory results reveal SCr 4.8 mg/dL, potassium 4.7<br />

mEq/L, AST 56 IU/L, and ALT 325 IU/L. His INR is 1.6,<br />

and he is scheduled for hemodialysis later today. Y.G.’s home<br />

drugs include amlodipine 10 mg/day, metoprolol 50 mg<br />

twice daily, a “kidney vitamin,” lactulose three times/day,<br />

clonidine 0.3 mg three times/day, and calcium acetate 667<br />

mg three times/day with meals. He has been actively listed<br />

for a kidney plus liver transplant for the past 5 months.<br />

PSAP 2013 • <strong>Cardiology</strong>/<strong>Endocrinology</strong> 21 Answers


52. Which one of the following best describes Y.G.’s stroke<br />

risk and bleeding risk according to the CHADS 2<br />

and<br />

HASBLED scores, respectively?<br />

A. Low, high.<br />

B. Intermediate, high.<br />

C. Intermediate, low.<br />

D. High, high.<br />

52. Answer: B<br />

The patient has one risk factor according to the CHADS 2<br />

score, which is hypertension. Therefore, his score is 1, and<br />

his risk of stroke would be considered intermediate. He has<br />

at least five risk factors for bleeding, including age older<br />

than 65, recent bleeding, abnormal liver function, abnormal<br />

kidney function, and history of hypertension, placing<br />

him at high risk of a bleeding event (Answer B is correct).<br />

Answer A, Answer C, and Answer D incorrectly assess the<br />

risk of stroke and/or bleeding.<br />

1. You JJ, Dinger DE, Howard PA, et al. Antithrombotic therapy<br />

for atrial fibrillation: antithrombotic therapy and<br />

prevention of therombosis, 9th edition: American College of<br />

Chest Physicians Evidence-Based Clinical Practice Guidelines.<br />

Chest 2012; 141(suppl 2); e531S-e575S.<br />

PubMed Link<br />

2. Gallego P, Roldan V, Torregrosa JM, et al. Relation of the HAS-<br />

BLED bleeding risk score to major bleeding, cardiovascular<br />

events, and mortality in anticoagulated patients with atrial<br />

fibrillation. Circ Arrhythm Electrophysiol 2012;5:312-8.<br />

PubMed Link<br />

53. According to the <strong>ACCP</strong> 2012 guidelines, which one of<br />

the following antithrombotic strategies is most appropriate<br />

for Y.G. to balance his risk of stroke and bleeding?<br />

A. Warfarin targeting an INR of 2.5-3.5.<br />

B. Aspirin 81 mg/day plus clopidogrel 75 mg/day.<br />

C. Aspirin 81 mg/day.<br />

D. No antithrombotic therapy indicated.<br />

53. Answer: C<br />

Given that the patient has an intermediate risk of stroke and<br />

a high risk of bleeding, Answer C likely represents the best<br />

answer according to the <strong>ACCP</strong> guidelines (Answer C is<br />

correct). Anticoagulation with warfarin would likely place<br />

this patient at high risk of a bleeding event, and the INR<br />

is higher than recommended for stroke prevention in atrial<br />

fibrillation (2.0-3.0) (Answer A is incorrect). Answer B is<br />

a reasonable for a patient with intermediate risk, but it is<br />

not recommended by <strong>ACCP</strong> if the reason for not choosing<br />

anticoagulation-based therapy is major bleeding risk given<br />

that aspirin plus clopidogrel did not significantly differ in<br />

major bleeding risk from warfarin in the ACTIVE study<br />

(Answer B is incorrect). Answer D is also not appropriate<br />

given that the patient is not at low risk of stroke and should<br />

receive some type of antithrombotic therapy (Answer D is<br />

incorrect).<br />

1. You JJ, Dinger DE, Howard PA, et al. Antithrombotic therapy<br />

for atrial fibrillation: antithrombotic therapy and<br />

prevention of therombosis, 9th edition: American College of<br />

Chest Physicians Evidence-Based Clinical Practice Guidelines.<br />

Chest 2012; 141(suppl 2); e531S-e575S.<br />

PubMed Link<br />

2. ACTIVE Writing Group of the ACTIVE Investigators.<br />

Clopidogrel plus aspirin versus oral anticoagulation for atrial<br />

fibrillation in the Atrial Fibrillation Clopidogrel Trial with<br />

Irbesartan for Prevention of Vascular Events (ACTIVE W): A<br />

randomised controlled trial. Lancet 2006;367:1903-12.<br />

PubMed Link<br />

54. After discussion, Y.G. expresses a wish to avoid ischemic<br />

stroke above other concerns, given his hope for<br />

a future kidney and liver transplant. Which one of the<br />

following represents the best anticoagulation strategy<br />

in Y.G., given his comorbidities and risk factors?<br />

A. Apixaban.<br />

B. Dabigatran.<br />

C. Rivaroxaban.<br />

D. Warfarin.<br />

54. Answer: D<br />

Given the patient’s end-stage chronic kidney disease, none<br />

of the newer anticoagulants can be safely recommended for<br />

use. Given the significant component of renal elimination<br />

for these agents, and the lack of established dosing paradigms<br />

in end-stage kidney disease, Answer A, Answer B,<br />

and Answer C are incorrect. Warfarin represents the best<br />

choice because of the ability to monitor and adjust therapy<br />

in response to the INR and because it is not excreted by<br />

the kidneys. The presence of moderate to severe liver dysfunction<br />

may also tilt the selection of anticoagulant therapy<br />

toward an agent in which therapeutic monitoring is possible<br />

(Answer D is correct).<br />

1. You JJ, Dinger DE, Howard PA, et al. Antithrombotic therapy<br />

for atrial fibrillation: antithrombotic therapy and<br />

prevention of therombosis, 9th edition: American College of<br />

Chest Physicians Evidence-Based Clinical Practice Guidelines.<br />

Chest 2012; 141(suppl 2); e531S-e575S.<br />

PubMed Link<br />

2. De Caterina R, Husted S, Wallentin L, et al. New oral anticoagulants<br />

in atrial fibrillation and acute coronary syndromes.<br />

ESC Working Group on Thrombosis—Task Force on<br />

Anticoagulants in Heart Disease position paper. J Am Coll<br />

Cardiol 2012;59:1413-25.<br />

PubMed Link<br />

Answers<br />

22<br />

PSAP 2013 • <strong>Cardiology</strong>/<strong>Endocrinology</strong>


55. Which one of the following is the most important factor<br />

in selecting an oral anticoagulant in Y.G., given<br />

near-term transplantation?<br />

A. Ability to monitor therapy.<br />

B. Drug-drug interactions.<br />

C. Reversibility.<br />

D. Drug acquisition cost.<br />

55. Answer: C<br />

In general, cadaveric transplantation requires emergency<br />

surgery to enhance graft survival and minimize organ ischemia<br />

time. Therefore, the reversibility of anticoagulation<br />

effect is a critical factor in drug selection (Answer C is correct).<br />

The ability to monitor therapy may be important, but<br />

that depends on the organ systems involved and the associated<br />

pharmacokinetic properties of the agent employed.<br />

Therefore, Answer A is incorrect as a general statement.<br />

Drug-drug interactions may also be significant, but this is<br />

likely to be after transplantation and is not a consideration<br />

for pretransplant drug selection (Answer B is incorrect).<br />

Acquisition cost can also be significant, but it is not the<br />

most critical factor in this scenario (Answer D is incorrect).<br />

1. You JJ, Dinger DE, Howard PA, et al. Antithrombotic therapy<br />

for atrial fibrillation: antithrombotic therapy and<br />

prevention of therombosis, 9th edition: American College of<br />

Chest Physicians Evidence-Based Clinical Practice Guidelines.<br />

Chest 2012; 141(suppl 2); e531S-e575S.<br />

PubMed Link<br />

2. De Caterina R, Husted S, Wallentin L, et al. New oral anticoagulants<br />

in atrial fibrillation and acute coronary syndromes.<br />

ESC Working Group on Thrombosis—Task Force on<br />

Anticoagulants in Heart Disease position paper. J Am Coll<br />

Cardiol 2012;59:1413-25.<br />

PubMed Link<br />

3. Wann LS, Curtis AB, January CT, et al. 2011 ACCF/AHA/<br />

HRS focused update on the management of patients with atrial<br />

fibrillation (updating the 2006 guideline). J Am Coll Cardiol<br />

2011;57:223-42.<br />

PubMed Link<br />

56. The ROCKET-AF trial showed that patients receiving<br />

rivaroxaban had a higher annual rate of stroke and<br />

systemic embolism compared with patients receiving<br />

dabigatran in the RE-LY trial (2.1% vs. 1.1% in<br />

the 150-mg group). Which one of the following statements<br />

most accurately describes the reason for this<br />

difference?<br />

A. Rivaroxaban is inferior to dabigatran given the<br />

mechanism of action.<br />

B. The RE-LY trial used a stronger clinical trial<br />

design.<br />

C. The ROCKET-AF trial enrolled higher-risk<br />

patients.<br />

D. Rivaroxaban is inferior to dabigatran when dosed<br />

once daily.<br />

56. Answer: C<br />

Patients in ROCKET-AF were only enrolled if they had two<br />

moderate risk factors for stroke or one major risk factor,<br />

whereas patients in RE-LY could be enrolled with only one<br />

risk factor for stroke. As a result, the mean CHADS 2<br />

score<br />

was 3.48 in the rivaroxaban group in ROCKET-AF compared<br />

with 2.2 in the dabigatran group in RE-LY (Answer C<br />

is correct). Answer A and Answer D are incorrect because<br />

it is difficult to make a head-to-head comparison between<br />

therapies across clinical trials that enrolled significantly different<br />

patient populations and used different clinical trial<br />

designs. Answer B is also incorrect; the double-dummy<br />

design used in ROCKET-AF is arguably a stronger design<br />

than the open-label program used in RE-LY.<br />

1. Patel MR, Mahaffey KW, Garg J, et al. Rivaroxaban versus<br />

warfarin in nonvalvular atrial fibrillation. N Engl J Med<br />

2011;365:883-91.<br />

PubMed Link<br />

2. Connolly SJ, Ezekowitz MD, Yusuf S, et al. Dabigatran versus<br />

warfarin in patients with atrial fibrillation. N Engl J Med<br />

2009;361:1139-51.<br />

PubMed Link<br />

Questions 57–59 pertain to the following case.<br />

Z.T. is a 60-year-old man admitted to the cardiology service<br />

after a non–ST-elevation myocardial infarction. He<br />

received one drug-eluting stent to the proximal left anterior<br />

descending coronary artery and was doing well the<br />

following day. He is a smoker (1 pack/day for 20 years)<br />

and regularly abuses alcohol. Three weeks ago he had an<br />

upper gastrointestinal bleed after an endoscopy coil embolization.<br />

He also has paroxysmal AF, currently in normal<br />

sinus rhythm. Z.T. is having asymptomatic runs of AF on<br />

telemetry. His current drugs include aspirin 81 mg/day,<br />

clopidogrel 75 mg/day, simvastatin 40 mg at bedtime, lisinopril<br />

5 mg/day, and an unfractionated heparin infusion at<br />

1200 units/hour. Z.T. was receiving warfarin before this<br />

hospitalization for his AF; on his arrival this was reversed<br />

with vitamin K and fresh frozen plasma. His BMP and CBC<br />

are within normal limits, his INR is 1.3, and his aPTT is 75<br />

seconds, which is therapeutic for heparin according to the<br />

hospital nomogram. After identification of a bruit on physical<br />

examination, he is also found to have a vascular fistula at<br />

the site of his catheterization.<br />

57. Which one of the following best describes Z.T.’s bleeding<br />

risk according to the HASBLED score?<br />

A. Very low.<br />

B. Low.<br />

C. Intermediate.<br />

D. High.<br />

PSAP 2013 • <strong>Cardiology</strong>/<strong>Endocrinology</strong> 23 Answers


57. Answer: D<br />

Z.T. has several risk factors for bleeding. He has a bleeding<br />

history or predisposition with his recent GI bleed and is currently<br />

receiving both dual antiplatelet therapy and actively<br />

abuses alcohol. His HASBLED score is calculated to be 3<br />

which is considered “high risk” and Answer D is therefore<br />

correct. Answer A is incorrect as this is not a recognized risk<br />

categorization for HASBLED. Answers B and C incorrectly<br />

calculate the risk score based on the patient specific factors.<br />

1. Lane DA, Lip GYH. Use of the CHA2DS2VASc and HAS-<br />

BLED scores to aid decision making for thromboprophylaxis<br />

in nonvalvular atrial fibrillation. Circulation 2012; 126: 860-5.<br />

PubMed Link<br />

2. Roldan V, Marin F, Fernandez H, et al. Predictive value of the<br />

HAS-BLED and ATRIA bleeding cosres for the risk of serious<br />

bleeding in a “real-world” population with atrial fibrillation<br />

receiving anticoagulant therapy. Chest 2013; 143 (1): 179-184.<br />

PubMed Link<br />

58. Considering his risk factors and other comorbidities,<br />

which one of the following would be best to recommend<br />

for Z.T. as antithrombotic therapy for AF?<br />

A. Continue aspirin and clopidogrel.<br />

B. Discontinue clopidogrel; start warfarin to target<br />

INR of 2.0–3.0.<br />

C. Discontinue clopidogrel; continue aspirin.<br />

D. Continue aspirin and clopidogrel; start warfarin<br />

to target INR of 2.0–3.0.<br />

58. Answer: A<br />

The patient’s risk of stroke is intermediate, according to<br />

CHA 2<br />

DS 2<br />

VASc, and he is at high risk of bleeding considering<br />

his drug abuse and recent gastrointestinal bleeding.<br />

Given this, the use of an anticoagulant to prevent stroke in<br />

AF is less preferable (Answer B is incorrect). Answer D is<br />

also incorrect, and the use of “triple therapy” (i.e., dual antiplatelet<br />

therapy plus an anticoagulant) is associated with a<br />

high risk of bleeding. In addition, his recent stent placement<br />

necessitates the use of dual antiplatelet therapy (Answer C<br />

is incorrect). Answer A is correct because the use of aspirin<br />

plus clopidogrel is recommended for stroke prevention<br />

in AF.<br />

1. Faxon DP, Eikelboom JW, Berger PB, et al. Antithrombotic<br />

therapy in patients with atrial fibrillation undergoing coronary<br />

stenting: a North American perspective: executive summary.<br />

Circ Cardiovasc Interv 2011;4:522-34.<br />

PubMed Link<br />

2. Lane DA, Lip GYH. Use of the CHA2DS2VASc and HAS-<br />

BLED scores to aid decision making for thromboprophylaxis<br />

in nonvalvular atrial fibrillation. Circulation 2012;126:860-5.<br />

PubMed Link<br />

59. Z.T.’s vascular fistula requires surgical correction.<br />

His antiplatelet therapy will continue given his recent<br />

stent. Which one of the following is the best statement<br />

regarding interruption of anticoagulation for<br />

stroke prevention in AF according to the ACC/AHA/<br />

HRS guidelines?<br />

A. Anticoagulation should not be interrupted for<br />

any time whatsoever.<br />

B. A short-term interruption (less than 1 week) is<br />

reasonable and requires no parenteral bridging.<br />

C. Parenteral anticoagulation should be reinitiated<br />

the day after surgery.<br />

D. A long-term interruption (greater than 1 month)<br />

is warranted given the clinical scenario.<br />

59. Answer: B<br />

The ACC/AHA/HRS guidelines recommend that a<br />

short-term interruption (less than 1 week) is reasonable<br />

in patients with AF (Answer B is correct). Answer A is<br />

incorrect given the patient’s impending surgery. Answer<br />

C is also incorrect given that anticoagulation soon after<br />

surgery may introduce a significant risk of bleeding and<br />

is unnecessary. Answer D is incorrect because it attributes<br />

the incorrect time frame to the ACC/AHA/HRS<br />

recommendation.<br />

1. Wann LS, Curtis AB, January CT, et al. 2011 ACCF/AHA/<br />

HRS focused update on the management of patients with<br />

atrial fibrillation (updating the 2006 guideline). J Am Coll<br />

Cardiol 2011;57:223-42.<br />

PubMed Link<br />

2. You JJ, Dinger DE, Howard PA, et al. Antithrombotic therapy<br />

for atrial fibrillation: antithrombotic therapy and prevention<br />

of therombosis, 9th edition: American College of Chest<br />

Physicians Evidence-Based Clinical Practice Guidelines.<br />

Chest 2012; 141(suppl 2); e531S-e575S.<br />

PubMed Link<br />

60. The following alerts to prescribers, nurses, or pharmacists<br />

are being considered for new oral anticoagulants<br />

used within an acute care health system. Which one<br />

of the following would best prevent an anticoagulation-related<br />

medication error?<br />

A. To prescribers that dabigatran results in<br />

significantly less intracerebral hemorrhage<br />

than warfarin when used as a stroke prevention<br />

strategy in AF.<br />

B. To pharmacists that display serum creatinine<br />

when warfarin is verified.<br />

C. To nurses that the dabigatran capsule should not<br />

be opened, chewed, or crushed.<br />

D. To nurses that dabigatran may cause dyspepsia.<br />

Answers<br />

24<br />

PSAP 2013 • <strong>Cardiology</strong>/<strong>Endocrinology</strong>


60. Answer: C<br />

The dabigatran capsule cannot be opened or manipulated<br />

in any way because doing so increases the bioavailability<br />

significantly and exposes the patient to a greater degree<br />

of anticoagulation than intended (Answer C is correct).<br />

Answer A may be a reasonable way to increase the use<br />

of dabigatran, but it is not a medication-error prevention<br />

strategy (Answer A is incorrect). Answer B would not be<br />

effective because warfarin use is not significantly affected<br />

by renal clearance (Answer B is incorrect). Answer D is<br />

also incorrect because, although it is useful to acknowledge<br />

potential adverse drug reactions, this is not likely to<br />

minimize the chance of a medication error.<br />

1. De Caterina R, Husted S, Wallentin L, et al. New oral anticoagulants<br />

in atrial fibrillation and acute coronary syndromes.<br />

ESC Working Group on Thrombosis—Task Force on<br />

Anticoagulants in Heart Disease position paper. J Am Coll<br />

Cardiol 2012; 59:1413-25.<br />

PubMed Link<br />

2. Wann LS, Curtis AB, January CT, et al. 2011 ACCF/AHA/<br />

HRS focused update on the management of patients with<br />

atrial fibrillation (updating the 2006 guideline). J Am Coll<br />

Cardiol 2011;57:223-242.<br />

PubMed Link<br />

PSAP 2013 • <strong>Cardiology</strong>/<strong>Endocrinology</strong> 25 Answers

Hooray! Your file is uploaded and ready to be published.

Saved successfully!

Ooh no, something went wrong!